ORTHOPEDIC MCQS ONLINE QUESTION BANK H3A

ORTHOPEDIC MCQS ONLINE QUESTION BANK H3A

 

 

1) 

 

 

3341. (2079) Q8-2501:

Indications for internal fixation of the humeral shaft include all of the following except:

 

1) Acute radial nerve palsy

3) Bilateral humeral fractures

2) Ipsilateral forearm fracture

5) Polytrauma patient

4) Worsening radial nerve palsy

 

An acute radial nerve palsy is not an indication for internal fixation of the humeral shaft. The majority of patients with acute radial nerve palsies spontaneously recover. A worsening radial nerve palsy is an indication for exploration; and a floating elbow in a polytrauma patient is an indication for internal fixation of the humeral shaft, as well.Correct Answer: Acute radial nerve palsy

 

 

3342. (2080) Q8-2502:

The primary goal of the Neer classification of proximal humeral fractures is to:

 

1) Determine the position of fragments

3) Delineate the number of fragments

2) Determine the best surgical approach

5) Determine whether the fracture has dislocated

4) Determine the vascularity of the articular segment

 

Neerâs four-part classification is designed to determine which fractures will result in avascular necrosis of the articular segment. In a four-part fracture, there is no residual blood supply to the articular segment.Correct Answer: Determine the vascularity of the articular segment

 

 

3343. (2081) Q8-2503:

Implants protruding beyond the medial humeral neck can impinge on which of the following structures:

 

1) Axillary nerve

3) Radial nerve

2) Axillary vein

5) Musculocutaneous nerve

4) Axillary artery

 

With internal rotation, any implant protruding beyond the medial cortex of the humeral neck can impinge on the main trunk of the axillary nerve as it courses behind the humerus.Correct Answer: Axillary nerve

 

 

 

1) Musculocutaneous nerve

3) Rotator cuff insertion

2) Suprascapular nerve

5) Deltoid muscle attachment

4) Axillary nerve

 

The axillary nerve and suprascapular nerve can be injured in the inferior and superior parts of the shoulder capsule. The deltoid attachment to the acromion can be damaged as well. These injuries can lead to disability. The musculocutaneous nerve is anterior and inferior and is an unlikely structure to be injured.Correct Answer: Musculocutaneous nerve

 

 

3345. (2083) Q8-2506:

Which modality is best for diagnosing stress fractures:

 

1) Computed tomography scanning

3) Magnetic resonance imaging

2) Plain radiographs

5) Clinical evaluation

4) Technetium bone scanning

 

Magnetic resonance imaging shows immediate classic appearance of stress fractures. Computed tomography scanning shows bony architecture that may take time to develop. Bone scanning is too nonspecific; it shows activity but is not diagnosti Radiographs sometimes take 2 to 3 weeks to become positive.Correct Answer: Magnetic resonance imaging

 

 

3346. (2084) Q8-2507:

Which of the following types of tibial fracture is most likely associated with compartment syndrome:

 

1) Closed

3) Grading of the fractures is irrevelant

2) Grade IIIA open

5) Grade IIIB open

4) Grade I or II open

 

A common fallacy is that the degree of open wound will prevent compartment syndrome by tearing the fascia. The amount of energy initially imparted to the tissues is the greatest determinant of a subsequent compartment syndrome.Correct Answer: Grade IIIB open

 

 

3347. (2085) Q8-2508:

In a compartment syndrome, pressure measurements are:

 

1) Highest at the fracture site

3) Highest closer to the outer fascia

2) Diffuse throughout the muscle belly

5) Highest at ends of the fascia

4) Elevated in surrounding compartments also

 

The pressure is the highest at the fracture sit This is true even though this is a hydraulic effect of pressure. As the needle is inserted farther away from the fracture, the pressure drops.Correct Answer: Highest at the fracture site

 

 

 

1) Intravenous cephalasporin antibiotics are required

3) Oral ciprofloxacin for 3 days is adequate

2) Intravenous cephalasporin and gentemicin antibiotics are required

5) Intravenous gentamicin alone is necessary

4) Antibiotics have no role

 

Oral ciprofloxacin twice a day for 3 days provides equal prophylaxis to all of the mentioned regimens.Correct Answer: Oral ciprofloxacin for 3 days is adequate

 

 

3349. (2087) Q8-2510:

Bullets must be removed if they are:

 

1) Intra-articular

3) In the peripheral muscles of the tibia

2) In the tissues of the thigh

5) Close to arteries

4) In the forearm muscles

 

Bullet fragments in the joint should be removed to prevent lead synovitis. If they are left in place, it could result in systemic lead toxicity.Correct Answer: Intra-articular

 

 

3350. (2088) Q8-2511:

Most low velocity gunshot wounds to the humerus can be successfully treated with:

 

1) Open reduction internal fixation with plates and screws

3) Locked intramedullary nailing

2) Flexible intramedullary nailing

5) Hanging arm casts

4) Functional bracing

 

Low velocity gunshot wounds do not significantly destabilize the soft tissue envelope. Functional bracing is efficacious in the treatment of these conditions with a reported 2% nonunion rate.Correct Answer: Functional bracing

 

 

3351. (2089) Q8-2512:

Isolated low velocity gunshot wounds to the femur are best managed by:

 

1) Immediate debridement and reamed locked intramedullary (IM) nailing

3) Plate fixation

2) Immediate skin edge debridement and early reamed locked IM nailing

5) External fixation followed by intramedullary nailing

4) Traction until the wound has healed followed by ream locked IM nailing

 

An isolated gunshot wound to the femur is not an immediate emergency. No significant increase in complications or infection results from waiting until the patientâs stomach is empty and a crew is available, provided the skin edges are debrided and a cephalosporin antibiotic is started.Correct Answer: Immediate skin edge debridement and early reamed locked IM nailing

 

 

3352. (2090) Q8-2513:

Which of the following is true concerning the blood supply to the soleus muscle:

 

  1. Arterial supply is entirely from the proximal muscle belly.

    3) There are no major arteries entering the muscle, only collaterals.

    2) Arterial supply is segmental.

    5) There is only one artery supplying the muscle

    1. The blood supply to the soleus muscle is inconsistent.

       

      There is a major artery entering the upper portion of the soleus, but the artery can be ligated and the soleus rotated based on its distal pedicle.Correct Answer: Arterial supply is segmental.

       

      3353. (2091) Q8-2514:

      The incidence of osteonecrosis in three-part fractures of the proximal humerus is approximately:

       

      1) 6%

      3) 43%

      2) 27%

    2. 68%

    4) 91%

     

    Three-part fractures of the proximal humerus usually represent a fracture of the surgical neck and a fracture of the greater tuberosity. Treatment results of these fractures are related to the residual tuberosity displacement and the angulation of the head relative to the neck.

     

    Osteonecrosis occurs in up to 27% of three-part fractures but is usually asymptomatic because it involves only a portion of the humeral head. Treatment, therefore, for these fractures should be based on the biomechanical aspects of the fracture rather than the vascularity of the humeral head.Correct Answer: 27%

     

     

    3354. (2092) Q8-2515:

    The incidence of radial nerve palsy in acute humeral shaft fractures in throwing athletes is approximately:

     

    1) 1%

    3) 16%

  2. 6%

    5) 56%

    4) 36%

     

    Ogawa and Yoshida analyzed 90 patients who sustained humeral shaft fractures while throwing to determine what caused their injuries. All patients were recreational baseball players, including 89 men and one woman. The average age was 25 years (range, 12 to 43 years). The throwing style, type of pitch, fielding position, and type of ball used varied; however, the patients sustained their fractures while performing a hard throw in 87 (97%) of the occurrences. The actual courses of the balls thrown ranged from sideways to straightforward. All fractures were external rotation spiral fractures; 25 patients (28%) had a medial butterfly fragment, and 14 patients (16%) had radial nerve palsy. Fractures were most likely to have occurred in the distal half of the humerus, although they occurred frequently in the proximal half in patients in their early teens.Correct Answer: 16%

     

     

     

    3355. (2093) Q8-2516:

    A 34-year-old man involved in a motor vehicle accident sustains a pneumothorax, a closed femur fracture, and closed, displaced fractures of the ipsilateral humerus, radius, and ulna. Management of his femur fracture is open reduction internal fixation.

    Management of his humerus fracture should consist of:

     

    1. Skeletal traction

  3. Open reduction and internal fixation of all fractures

    1. Closed reduction and plaster immobilization

      5) Internal fixation of the humeral fracture and immobilization of the forearm fracture

      1. External fixation of all fractures

         

        Indications for open reduction and internal fixation of humeral shaft fractures include open fractures, vascular injuries, floating elbow, polytrauma with multiple extremities, closed-head injury, and pathologic fractures.Correct Answer: Open reduction and internal fixation of all fractures

         

        The incidence of fractures of the coronoid process in acute elbow dislocations is approximately:

         

        1) 1%

        3) 10%

        2) 5%

      2. 40%

  4. 20%

     

    Coronoid fractures are common and are associated with 10% of elbow dislocations and recurrent instability after dislocation. A review of 35 patients who had a fracture of the coronoid process of the ulna revealed three types of fracture: type I â avulsion of the tip of the process; type II â a fragment involving 50% of the process or less; and type III â a fragment involving more than 50% of the process. A concurrent dislocation or associated fracture was present in 14%, 56%, and 80% of these patients, respectively.

     

    The outcome correlated well with the type of fracture. According to an objective elbow-performance index used to assess the results for the 32 patients who had at least 1 year of follow-up (mean, 50 months), 92% of the patients who had a type I fracture, 73% who had a type II fracture, and 20% who had a type III fracture satisfactory results. Residual stiffness of the joint was most often present in patients who had a type III fracture.Correct Answer: 10%

     

     

    3357. (2095) Q8-2518:

    After a fall, a 30-year-old man presents to the emergency department complaining of elbow pain. Examination in the emergency department reveals a two-part displaced radial head fracture with no comminution. The next step in management should include:

     

    1) Open reduction internal fixation

    3) Long arm cast

    2) External fixation

    5) Radial head excision

    4) Examination with local anesthesia to assess elbow motion

     

    Opinions are diverse regarding the approach to management of two-part displaced radial head fractures. This fracture pattern can vary in the amount of articular surface involved, the extent of impaction of the fracture fragment, the presence or absence of comminution in the radial neck, and the associated soft tissue involvement. It is of fundamental importance to assess the amount of forearm rotation and elbow flexion. When no block to motion exists and there is no palpable incongruity of the articular surface, conservative treatment with a splint provides a good result. If, however, a definable block to motion is demonstrated, operative intervention should be considered. At this point, the presence of associated soft tissue or skeletal trauma becomes an essential factor in the decision-making process. Examination under anesthesia, examination after intra-articular injection of local anesthetic, and stress radiographs are helpful in establishing the diagnosis.Correct Answer: Examination with local anesthesia to assess elbow motion

     

     

     

    3358. (2096) Q8-2519:

    Which of the following motions is likely to be most affected after healing of a displaced greater tuberosity fracture:

     

    1) Elevation

    3) Extension

    2) Abduction

    5) External rotation

    4) Internal rotation

     

    Studies involving malunion of the greater tuberosity and malposition of the tuberosity from a varus surgical neck malunion indicate a need to evaluate these lesions more carefully. The limit of 1-cm displacement or 45° angulation may be too generous for classifying the lesion as nondisplaced, especially when the tuberosity assumes a position superior to the articular surface when the arm is resting at the side. On elevation, the normally small clearance in the subacromial space, if compromised, leads to impingement and tearing of the rotator cuff.Correct Answer: Elevation

     

    For comminuted fractures of the radial head with more than three articular fragments (Mason type 3), the preferred treatment is:

     

    1) Early range of motion

    3) External fixation

    2) Open reduction internal fixation

    5) Total elbow arthroplasty

    4) Excision of the radial head with or without prosthetic replacement

     

    Ring and associates examined 56 patients with intra-articular fractures of the radial head who had been treated with open reduction and internal fixation. Patients were evaluated at an average of 48 months after injury. Thirty patients had a Mason type 2 (partial articular) fracture, and 26 patients had a Mason type 3 (complete articular) fracture. The result at the final evaluation was judged to be unsatisfactory when there was early failure of fixation or nonunion, a second operation was required to excise the radial head, less than 100° of forearm rotation was present, or the patient received a fair or poor rating according to the system of Broberg and Morrey.

     

    Thirteen of the 14 patients with a Mason type 3 comminuted fracture with more than three articular fragments had an unsatisfactory result. Of the twelve patients with a type 3 fracture that split the radial head into two or three simple fragments, none had early failure, one had nonunion, and all had an arc of forearm rotation of greater than or equal to 100°. The data suggest that open reduction and internal fixation is best reserved for minimally comminuted fractures with three or fewer articular fragments. Comminuted fractures and fractures associated with complex elbow or forearm dislocation may be better treated by radial head excision with or without prosthetic replacement.Correct Answer: Excision of the radial head with or without prosthetic replacement

     

     

     

    3360. (2098) Q8-2521:

    Which of the following mechanisms is the most common in cervical spine injuries in football players:

     

    1) Rotation

    3) Hyperextension

    2) Hyperflexion

    5) Distraction

    4) Axial loading

     

    The National Football Head and Neck Injury Registry has compiled epidemiologic, pathologic, biomechanical, and cinematographic data on head and neck injuries occurring in tackle football since 1971. Preliminary analysis performed in 1975 indicated that axial loading caused the majority of serious cervical spine football injuries.Correct Answer: Axial loading

     

     

    3361. (2099) Q8-2522:

    Which of the following solutions is appropriate for the initial fluid management in a severely hypotensive trauma patient:

     

    1) Saline, 1-L bolus

    3) Saline, 1-L over a 1-hour period

    2) Ringerâs lactate, 1-L bolus

    5) Ringerâs lactate, 2-L bolus

    4) Ringerâs lactate, 1-L over a 1-hour period

     

    Infusion of a balanced salt solution such as Ringerâs lactate or saline should be started. Hypertonic saline solutions and colloid-containing solutions have also been used in trauma resuscitation protocols with varying degrees of success. The severely hypotensive trauma patient in hypovolemic shock should receive 2-L of a balanced salt solution as rapidly as possible.Correct Answer: Ringerâs lactate, 2-L bolus

     

    The maximum amount of sagittal plane deformity that is acceptable for a fracture in the middle third of the proximal phalanx is:

     

    1) 5°

    3) 20°

    2) 10°

    5) 40°

    4) 30°

     

    Nondisplaced and minimally displaced fractures of the phalangeal shaft tend to be stable and can be statically immobilized or treated in dynamic fracture casts for 3 or 4 weeks. Fractures rarely displace more than that seen on the initial radiograph, however, they should be serially monitored for position, alignment, and healing. A guideline for acceptable reduction is no more than 10° of angulation in any plane of proximal and middle phalangeal shaft fractures and no clinical rotational malalignment.Correct Answer: 10°

     

     

    3363. (2101) Q8-2524:

    Which of the following is the most significant risk factor for the development of adult respiratory distress syndrome in the multiply injured patient with a femur fracture:

     

    1) Chest injury

    3) Head injury

    2) Delay of 18 hours in stabilizing the fracture

    5) Abdominal injury requiring laparotomy

    4) Use of a reamed intramedullary rod for femoral fixation

     

    A retrospective review of patients with multiple injuries with Injury Severity Scores of 18 points or higher who also had severe chest trauma was performed at the Erie County Medical Center in Buffalo, New York. Patients were divided into three groups. The patients in group 1 had femoral fractures that were treated with early stabilization with a reamed rod. The patients in group 2 had femoral fractures stabilized with plate fixation. In group 3, there were no femoral fractures. The Injury Severity Score for each group was nearly the same; however, the time in the intensive care unit for group 2 and group 3 was more than double that in group 1. There was a 33% adult respiratory distress syndrome rate in group 2, a 27% rate in group 3, and a 0% rate in group 1. The mortality rate was 10.9% in group 3 (patients without femoral fractures), which was the highest mortality rate among the three groups. These clinical data seem to associate a chest injury with increased rates of adult respiratory distress syndrome and mortality, as opposed to the method of treatment for the femoral fracture.Correct Answer: Chest injury

     

     

     

    3364. (2102) Q8-2525:

    Which of the following tibial fractures is most likely to have residual angulation (more than 5°) after treatment with a statically locked intramedullary rod:

     

    1) A distal third oblique fracture

    3) A comminuted midshaft fracture

    2) A proximal third metaphyseal fracture

    5) An open transverse fracture with a large butterfly fragment

    4) A transverse midshaft fracture

     

    Fractures of the proximal third of the tibial shaft do not appear to respond as favorably to intramedullary nailing as do fractures in the distal two-thirds of the tibia. Valgus, apex anterior angulation, and residual displacement at the fracture site are common after nailing. Surgical errors of a medialized nail entry point and a posteriorly and laterally directed nail insertion angle contributed to malalignment. Based on their findings, the authors have limited the use of intramedullary nailing for proximal third tibial shaft fractures and consider alternate forms of fixation (plate or external fixation). An alternative technique is the use of blocking screws.Correct Answer: A proximal third metaphyseal fracture

     

     

    Which of the following tibial plateau fractures is often associated with vascular injury:

     

    1) Schatzker type I

    3) Schatzker type III

    2) Schatzker type II

    5) Bilateral fractures

    1. Schatzker type IV

       

      A Schatzker type IV fracture is a fracture of the medial tibial plateau and may be a split or a split depression fracture. Many of these fractures represent a medial dislocation of the knee that has been reduced by the time the radiographs are taken. It is not the fracture of the medial plateau that gives this fracture its bad prognosis, but the associated injuries to the popliteal artery and peroneal nerve, as well as injuries to the collateral and cruciate ligaments. The arterial injury may be a rupture or only an intimal tear. Because of the frequence of associated popliteal artery injuries, whenever this lesion is recognized patients should be considered for an arteriogram to evaluate the artery and prevent and intraoperative or postoperative thrombosis.Correct Answer: Schatzker type IV

       

       

       

      3366. (2104) Q8-2527:

      The mortality rate associated with open pelvic fractures is approximately:

       

      1) 10%

      3) 30%

      2) 20%

    2. 75%

    4) 50%

     

    The mortality rate among 604 patients with pelvic fractures was 12%. Thirty-one percent had an open pelvic fracture, which is an injury associated with a 50% mortality rate. Twenty-three percent had pelvic fractureârelated iliac or femoral vessel disruptions, which are injuries associated with a 75% mortality rate. Mortality in these patients clearly resulted from ineffective control of pelvic hemorrhage and the inability to prevent sepsis in the pelvic hematoma.Correct Answer: 50%

     

     

    3367. (2105) Q8-2528:

    All of the following are indications for surgical management of humeral shaft fractures except:

     

    1. Floating elbow

    3) Varus angulation of 10°

    2) Associated brachial plexus palsy

    5) Polytrauma

    4) Bilateral humeral fractures

     

    Polytraumatized patients benefit from internal fixation of their humeral fracture to facilitate comfort, mobility, and nursing care. Bilateral humeral fractures constitute a specific injury complex in which humeral stabilization presents distinct advantages, allowing the patient earlier recovery of independence. Humeral fractures that are associated with both-bone forearm fractures, that is the "floating elbow" injury, require internal fixation to prevent nonunion and allow an earlier functional recovery of the entire limb. Patients with a brachial plexus palsy lose important muscular support of the fracture because of their motor dysfunction, which often results in distraction of the fracture fragments. Nonoperative treatment of such a fracture carries a high risk of nonunion, and internal fixation with a compression plate has been shown to improve results. The clinician should not accept more than 15° of varus angulation unless the patient is willing to accept visible deformity after union.Correct Answer: Varus angulation of 10°

     

     

     

    3368. (2106) Q8-2529:

    The incidence of complications and poor outcomes is high in fractures with vascular injury. Treatment for fractures with vascular injury should begin with which of the following steps:

     

    1. Fracture alignment and stability

      3) Recognition of compartment syndrome

      2) Wound management

      5) Early mobilization

      1. Early (within 6 hours of injury) restoration of blood flow

         

        The most important factor in treating fractures with a vascular injury is early restoration of blood flow. If provisional fracture stabilization precludes restoration of blood flow within this time frame, then arterial repair or placement of a temporary shunt should proceed first.Correct Answer: Early (within 6 hours of injury) restoration of blood flow

         

        3369. (2107) Q8-2530:

        What percentage of patients sustaining ankle sprains have residual dysfunction at 6 months:

         

        1) 10%

        3) 40%

        2) 20%

      2. 80%

      4) 60%

       

      At the United States Military Academy, all cadets presenting with ankle injuries during a 2-month period were included in this prospective, observational study. The initial evaluation included an extensive questionnaire, physical examination, and radiographs. There were 104 ankle injuries accounting for 23% of all injuries seen. There were 96 sprains, 7 fractures, and 1 contusion.

       

      At 6 months, all subjects had returned to full activity; however, 40% of cadets reported residual symptoms and 2.5% had a decrement of more than 20% on the lateral hop test. The factor most predictive of residual symptoms was a syndesmosis sprain, regardless of grade. Syndesmosis sprains were most prevalent in collision sports.

       

      This study demonstrates that even though our knowledge and understanding of ankle sprains and rehabilitation of these injuries have progressed in the last 20 years, chronic ankle dysfunction continues to be a prevalent problem. The early return to sports occurs after almost every ankle sprain; however, dysfunction persists in 40% of patients for as long as 6 months after injury.

      Syndesmosis sprains are more common than previously thought, and this confirms that syndesmosis sprains are associated with prolonged disability.Correct Answer: 40%

       

       

      3370. (2108) Q8-2531:

      In what percentage of ankle fractures does arthroscopically proven articular cartilage damage to the talus occur:

       

      1) 10%

      1. 50%

    2. 20%

  5. 100%

  1. 70%

     

    We have prospectively evaluated the arthroscopic findings in acute fractures of the ankle in 288 consecutive patients (148 men and 140 women) with a mean age of 45.6 years. According to the AO Danis-Weber classification, there were 14 type A fractures, 198 type B, and 76 type C. Lesions of the cartilage were found in 228 ankles (79.2%). These lesions were more often on the talus (69.4%) than on the distal tibia (45.8%), the fibula (45.1%), or the medial malleolus (41.3%).Correct Answer: 70%

     

     

    3371. (2109) Q8-2532:

    The most common thoracic injury after blunt trauma is:

     

    1) Rib fracture

    3) Myocardial contusion

    2) Aortic rupture

    5) Pneumothorax

    4) Pulmonary contusion

     

    Thoracic trauma occurs in 50% to 75% of all fatalities caused by blunt trauma. However, only 15% of thoracic injuries require surgical intervention. After head injuries, thoracic injuries are the second leading cause of death in trauma patients. Rib fractures are the most common (40%) thoracic injury in blunt trauma. Associated injuries (such as aortic arch rupture) should be suspected with high rib fractures. Liver and spleen injuries are common with low rib injuries. Pulmonary and cardiac contusion should be suspected in multiple rib injures.Correct Answer: Rib fracture

     

    Immediate stabilization of femur fractures within the first 24 hours in polytrauma patients decreases the risk of which of the following:

     

    1) Infection

    3) Heterotopic ossification

    2) Nonunion

    5) Pulmonary complications

    4) Shortening

     

    The patients were divided into two groups: patients who had an isolated fracture of the femur and those who had multiple injuries. When stabilization of the fracture was delayed in the patients who had multiple injuries, the incidence of pulmonary complications (adult respiratory distress syndrome, fat embolism, and pneumonia) was higher, the hospital stay was longer, and the number of days in the intensive care unit was increased.Correct Answer: Pulmonary complications

     

     

    3373. (2111) Q8-2534:

    The risk for avascular necrosis of the hip following stabilization of femur fractures with ipsilateral femoral neck and shaft fractures is:

     

    1) 40%

    1. Less than 5%

      2) 60%

      5) 25%

    2. 10%

     

    A midshaft femoral fracture in a high-energy trauma victim should prompt the orthopedist to search carefully for an ipsilateral fracture of the femoral neck. This fracture is present in 5% of patients, and the incidence may be increasing. Management of the femoral neck fracture should include anatomic reduction and screw fixation with compression. The use of capsulotomy remains controversial. Management of the femoral shaft should not interfere with these goals because of the risk of post-traumatic necrosis of the femoral head. The risk of avascular necrosis after stabilization of ipsilateral femoral neck and shaft fractures is less than 5%. Successful methods of management of the femoral shaft fracture have included compression plating, retrograde Kuntscher nailing, and interlocking nails. Approaches to the treatment of concomitant intertrochanteric and femoral shaft fractures should be selected according to the skill and experience of the surgeon and the availability of equipment.Correct Answer: Less than 5%

     

     

     

    3374. (2112) Q8-2535:

    The percentage of missed femoral neck fractures in fractures with an ipsilateral femoral neck and shaft fractures is approximately:

     

    1. Less than 5%

      3) 35%

    2. 20%

  2. 75%

4) 50%

 

Ipsilateral fractures of the femoral neck and shaft present diagnostic difficulties and complex choices with regard to treatment. A review of the 83 cases reported in the literature revealed that approximately one-third of the fractures of the femoral neck were missed initially.Correct Answer: 35%

 

 

3375. (2113) Q8-2536:

Posterior shoulder dislocations are most often associated with which of the following fractures:

 

  1. Lesser tuberosity

3) Greater tuberosity

2) Clavicle

5) Surgical neck

4) Scapular neck

 

When isolated two-part lesser tuberosity fractures occur, a posterior dislocation should be taken for granted until accurate imaging studies can disprove it. An axillary lateral view radiograph or computed tomography scan will establish the diagnosis.Correct Answer: Lesser tuberosity

 

Which of the following fractures occur in up to one-third of anterior shoulder dislocations and is the most common isolated fracture occurring in skierâs:

 

1) Lesser tuberosity fractures

3) Clavicle fractures

2) Scapular neck fractures

5) Surgical neck fractures

4) Greater tuberosity fractures

 

Two-part greater tuberosity fractures comprise 14% of displaced proximal humeral fractures with or without anterior dislocation and are the most common isolated fractures occurring in skiers. Anatomically, there are separate facets for the supraspinatus, infraspinatus, and teres minor insertions. Significant displacement of any or all of these facets is pathognomonic of a longitudinal tear of the rotator cuff. The indications for treatment include displacement greater than 5 mm. Greater tuberosity fractures may occur in up to one-third of anterior shoulder dislocations. The glenohumeral joint frequently reduces spontaneously before emergency treatment and radiographic documentation.Correct Answer: Greater tuberosity fractures

 

 

3377. (2115) Q8-2538:

The method of stabilization of open tibial fractures should be based primarily on:

 

1) Age of the patient

3) Soft tissue injury

2) Fracture pattern

5) Severity of other musculoskeletal injuries

4) Amount of bone loss

 

As classification systems have become more focused in differentiating severe tibia fractures, it has become apparent that soft tissue damage, including periosteal stripping and tissue loss, is the primary determinant of infection and nonunion. Meticulous wound care is the cornerstone of successful management of all open fractures of the tibial shaft, regardless of the implant used for tibial fixation.Correct Answer: Soft tissue injury

 

 

3378. (2116) Q8-2539:

The safest distance below the knee for placement of external fixation wires is:

 

1) 5 mm

3) 14 mm

2) 9 mm

5) 25 mm

4) 19 mm

 

The joint capsule reflects 14 mm below the joint line. Pins placed less than 14 mm below the knee are at risk for articular penetration. Violation of the proximal tibio-fibular joint can also lead to knee sepsis.Correct Answer: 14 mm

 

 

3379. (4050) Q8-2540:

When comparing retrograde femoral nailing to antegrade femoral nailing, which of the following statements is true:

 

1) Both femoral nailing techniques have similar rates of union.

3) Antegrade femoral nailing has a higher rate of union.

2) Retrograde femoral nailing has a higher rate of malunion.

5) Antegrade femoral nailing is always superior.

4) Retrograde femoral nailing is always superior.

 

When considering the entire length of the femur with similar reaming techniques, there are no differences in union rates, malunion rates, or times to union.Correct Answer: Both femoral nailing techniques have similar rates of union.

 

The greatest difference between antegrade femoral nailing and retrograde femoral nailing is:

 

1) Infection rate

3) Hip versus knee pain

2) Distal alignment

5) Union rates

4) Iatrogenic fractures

 

The entry portal damage determines the difference between antegrade femoral nailing and retrograde femoral nailing. Antegrade femoral nailing damages the hip, whereas retrograde femoral nailing damages the knee. The relative severity of damage has not been determined.Correct Answer: Hip versus knee pain

 

 

3381. (2118) Q8-2542:

When comparing reamed femoral nails to nonreamed femoral nails:

 

1) Reamed femoral nails unite faster than nonreamed femoral nails.

3) There is no difference in union time between reamed femoral nails and nonreamed femoral nails.

2) Nonreamed femoral nails unite faster than reamed femoral nails.

5) Nonreamed nails protect the lungs.

  1. The rate of nonunion is higher in patients with nonreamed femoral nails.

     

    When comparing patients with reamed femoral nails to patients with nonreamed femoral nails, both groups of patients will reliably unite but reamed femoral nails unite faster â 80 days versus 109 days.Correct Answer: Reamed femoral nails unite faster than nonreamed femoral nails.

     

     

    3382. (2119) Q8-2543:

    When using nonreamed tibial nails in blunt polytrauma patients, the delayed and nonunion rates are:

     

    1) 15%

    3) 39%

    2) 27%

  2. 60%

4) 51%

 

In a study by Riemer and colleagues, a reoperation rate of 39% to achieve union was reported. Therefore, some authors have advocated routine dynamization.Correct Answer: 39%

 

 

3383. (2120) Q8-2544:

When comparing titanium nonreamed tibial nails to stainless steel nonreamed tibial nails:

 

  1. There is no clinical difference between titanium nonreamed tibial nails and stainless steel nonreamed tibial nails.

3) Titanium nonreamed tibial nails frequently fail due to notching from locking screws.

2) Titanium nonreamed tibial nails unite faster than stainless steel nonreamed tibial nails.

5) The nail determines union rates.

4) Stainless steel nonreamed tibial nails and locking screws fail from fatigue more frequently.

 

The high failure rate (25%) of stainless steel nonreamed tibial nails is due to fatigue. Titanium nonreamed tibial nails have a lower failure rate (2%) as a result of fatigue.Correct Answer: Stainless steel nonreamed tibial nails and locking screws fail from fatigue more frequently.

 

When inserting nonreamed nails in an acute tibial diaphyseal fracture, the most important prognostic factor that predicts union is:

 

1) Grading of open fractures

3) Longitudinal localization

2) Classification of comminution

5) Patient age

4) Locking mode

 

The locking mode used in the tibial diaphysis (unlike the femur) has a dramatic effect on union rate and time.Correct Answer: Locking mode

 

 

3385. (2122) Q8-2546:

Anterior articular marginal fractures of the distal radius are:

 

1) Best managed by compression plating

3) Usually treated by buttress plating

2) Usually amenable to closed reduction

5) Well managed by percutaneous pins

4) Well managed by external fixation

 

An anterior articular marginal fracture of the distal radius requires buttress plating. Adequate fixation may be attained without screws in the distal fragment. Closed reductions are rarely successful. Locked plating is inadequate.Correct Answer: Usually treated by buttress plating

 

 

3386. (2123) Q8-2547:

Which of the following statements regarding muscle pedicle grafts for acute femoral neck fractures is true:

 

1) The infection rate is 20%.

3) Positioning of muscle pedicle grafts for acute femoral neck fractures is difficult.

2) Operating room time is excessive.

5) Some complications rates may result from fixation in the lateral position on a fracture table.

4) The incidence of hardware failure is identical when using muscle pedicle grafts.

 

The use of muscle pedicle grafts for acute femoral neck fractures is associated with high complication rates, totaling 68% (implant failure rate of 48%, an infection rate of 20%, and excessive operating room time). Some of the complication rates may result from fixation in the prone position on a fracture table.Correct Answer: Operating room time is excessive.

 

 

3387. (2124) Q8-2548:

Fibular fixation in comminuted pilon fractures:

 

1) Is always necessary

3) Is safe and reliable

2) Is best accomplished with intramedullary fixation

5) Should never be a consideration

4) Is associated with a high complication rate with questionable benefit when spanning the joint with a unilateral hinged fixator

 

Adequate ligamentotaxis allows acceptable results. Fixation is associated with a high complication rate including infection, nonunion, and malunion.Correct Answer: Is associated with a high complication rate with questionable benefit when spanning the joint with a unilateral hinged fixator

 

Reudi & Allgrower have shown that plate and screw ORIF for pilon gives excellent results if all of the follwing are followed except:

 

1) Anatomic reconsturction of the joint obtained

3) Bond graft

2) Good rigid internal fixation

5) Non weight bearing for 3 months

4) External fixation

 

Reviewing the radiographs of this period it was clear that insufficient reconstruction of the joint was obtained and this, in addition to less rigid fixation and the absence of a bone-graft, may have been responsible for the poor results. In addition to the importance of good reconstruction of the ankle-joint we would like to stress the importance of relieving the leg of weight-bearing for a minimum period of 3 months after operation but to combine this with active movement.Correct Answer: External fixation

 

 

3389. (2126) Q8-2550:

In an animal model, femoral intramedullary nail fixation in the presence of a pulmonary injury and adequate resuscitation:

 

1) Has a negative effect on pulmonary function

3) Has no effect on pulmonary function

2) Has a delayed effect on pulmonary function

5) Is not associated with dissemination of intravascular fat

  1. Is associated with a disseminated intravascular-type state

     

    With adequate resuscitation, reaming the femur does not jeopardize pulmonary function although increased fat and intraluminal pressures are present.Correct Answer: Has no effect on pulmonary function

     

     

    3390. (2127) Q8-2551:

    When using small wire fixators for periarticular fractures the incidence of septic arthritis is approximately:

     

    1) 2%

    3) 15%

    2) 10%

  2. 30%

4) 25%

 

A higher incidence of septic arthritis is found in plateau fractures compared to pilon fractures, but the overall rate for septic arthritis is approximately 2%.Correct Answer: 2%

 

 

3391. (2128) Q8-2552:

Reamed retrograde femoral nailing:

 

  1. Increases the vascularity of the anterior cruciate ligament

3) Has no effect on the vascularity of the knee ligaments

2) Decreases the vascularity of the posterior cruciate ligament

5) Alters the flow of blood to the entire leg

4) Increases the vascularity of the posterior cruciate ligament

 

Reamed retrograde nailing of the femur through the knee significantly decreases the blood flow through both cruciate ligaments by approximately 50%. The clinical significance of this finding is unclear.Correct Answer: Decreases the vascularity of the posterior cruciate ligament

 

Which of the following is a contraindication to reamed nailing for tibial nonunions:

 

1) Previous sepsis

3) Angular deformity

2) Translational deformity

5) Fibrous union

4) Rotational deformity

 

The relative contraindications to reamed nailing for tibial nonunions are shortening and previous sepsis. Most other deformities can be corrected and nailed.Correct Answer: Previous sepsis

 

 

3393. (2130) Q8-2554:

Disadvantages of Hackethal retrograde humeral nailing include:

 

1) Shoulder stiffness

3) Elbow stiffness

2) Painful hardware

5) Nonunion

4) Infection

 

Most patients treated with Hackethal retrograde humeral nailing for humeral shaft fractures require a second procedure for implant removal. Elbow stiffness and irritation are transient.Correct Answer: Painful hardware

 

 

3394. (2131) Q8-2555:

Which of the following statements is false when comparing plating to Hackethal retrograde nailing for humeral diaphyseal fractures:

 

1) Similar rates of joint function are achieved with both techniques.

3) Hackethal nails have a higher union rate than plating.

2) Nails require removal, not plates.

5) Nails require the support of a brace.

4) Similar union rates are achieved with both techniques.

 

Other than the need for metal removal and the support of a brace, Hackethal nailing and plating produce similar results.Correct Answer: Hackethal nails have a higher union rate than plating.

 

 

3395. (2132) Q8-2556:

In tibial shaft fractures with an arterial injury below the popliteal artery:

 

1) One patent artery is not always adequate for limb survival.

3) Bony injury does not correlate with poor clinical results.

2) Delay to revascularization and degree of soft tissue injury correlate with amputation rates.

5) Intramedullary nailing is mandatory.

4) Amputation is inevitable due to the size of the vessels.

 

In patients with an arterial injury below the popliteal artery, the period of avascularity and the soft tissue injury determine the amputation rate. Among those patients who keep their limbs, the bony injury correlates with function.Correct Answer: Delay to revascularization and degree of soft tissue injury correlate with amputation rates.

 

What factor most accurately determines functional outcomes after adequately reducing closed low-energy fractures of the tibial plateau:

 

1) Fracture classification

3) Age

2) Further reduction of the fracture

5) Activity level

4) Mechanism of injury

 

After an adequate reduction is attained in relatively low-energy plateau fractures, age determines functional outcome. According to a study by Stevens and colleagues, 40 years of age is the reported cut-off for determining functional outcome. This study points out the importance of age in prognosis.Correct Answer: Age

 

 

3397. (2134) Q8-2558:

Nonreamed intramedullary nailing of the tibia is superior to external fixation in all parameters except:

 

1) Union rates

3) Surgical site infection rates

2) Traumatic site infection rates

5) Reoperation rates

4) Rates of malalignment

 

External fixation and nonreamed intramedullary nailing are associated with equivalent rates of union.Correct Answer: Union rates

 

 

3398. (2135) Q8-2559:

When comparing antegrade nailing to plating of the humerus, which of the following statements is false:

 

1) Identical union rates are achieved with both antegrade nailing and plating of the humerus.

3) Plating of the humerus is associated with more elbow complications than antegrade nailing.

2) Antegrade nailing is associated with more shoulder complications than plating of the humerus.

5) Higher union rates result with plating of the humerus.

4) Identical rates of neurologic injuries are seen.

 

Chapman and associates demonstrated that antegrade nailing and plating of the humerus were the same, trading shoulder for elbow complications. Nailing was also associated with a higher rate of metal removals.Correct Answer: Higher union rates result with plating of the humerus.

 

 

3399. (2136) Q8-2560:

The most important factor in predicting cutout of an implant to repair intertrochanteric fractures of the hip is:

 

1) Size of the chosen screw

3) Tip/apex distance

2) Posterior/inferior placement

5) Angle of the plate

4) Pitch of the chosen screw

 

The most important factor in predicting cutout of an implant is the tip/apex distance. Screw design is a consideration but is limited by the anatomy of the proximal femur. Posterior/inferior placement was a factor used with old devices that did not purchase adequately in the femoral head.Correct Answer: Tip/apex distance

 

Which treatment should be avoided when repairing a tibial nonunion after external fixation:

 

1) Nonreamed nailing

3) Reamed nailing

2) Bone grafting

5) Plating

4) Electrical stimulation

 

Reamed nailing after tibial external fixation is associated with a high level of infection and persistent nonunion. Nonreamed nailing has proven far more efficacious. Electrical stimulation and bone grafting may also be used. Plating has been shown to be efficacious in certain circumstances.Correct Answer: Reamed nailing

 

 

3401. (2138) Q8-2563:

Neurologic function in compartment syndromes:

 

1) Deteriorates early

3) Is rarely compromised

2) Is necessary to diagnose a compartment syndrome

5) Is determined by the vascularity of the nerves within the specific compartment

4) Always deteriorates

 

Compartment syndromes involve the muscles in a compartment. Pressure can inconsistently effect nerves. Some compartments have no major nerves and are never associated with neurologic symptoms.Correct Answer: Is determined by the vascularity of the nerves within the specific compartment

 

 

3402. (2139) Q8-2564:

Reamed humeral nailing is associated with high rates of:

 

1) Iatrogenic comminution

3) Infection

2) Persistent nonunion after exchange

5) Reaming the humerus is irrelevant.

  1. Nonunion

     

    Reaming of the humerus should be avoided because the canal is tight and tapers distally. The bone is hard and cannot be reliably cut without excessive heat generation.Correct Answer: Reaming the humerus is irrelevant.

     

     

    3403. (2140) Q8-2565:

    The incidence of femoral shaft fractures that have an associated femoral neck fracture is approximately:

     

    1) 1%

    3) 6%

    2) 3%

  2. 15%

4) 10%

 

Approximately 6% of femoral shaft fractures caused from blunt trauma have an associated femoral neck fracture.Correct Answer: 6%

 

 

 

  1. An entry portal too medial during antegrade nailing

3) An entry portal too anterior during antegrade nailing

2) Unrelated to the antegrade nailing technique

5) Usually related to a technical error

4) Excessive force when inserting a nail

 

The late discovery of femoral neck fracture is unrelated to the antegrade nailing technique. Similar complications are reported with femoral plating when the neck is not instrumented. These complications always relate to the original trauma.Correct Answer: Unrelated to the antegrade nailing technique

 

 

3405. (2142) Q8-2567:

When comparing reamed and nonreamed nailing to reconstruct the tibial diaphysis after external fixation, which of the following statements is true:

 

1) Reamed nails heal faster than nonreamed nails.

3) There is no difference between reamed and nonreamed nailing to reconstruct the tibial diaphysis after external fixation.

2) Previous pin tract infections are irrelevant.

5) Nonreamed nails are capable of stimulating union more reliably than reamed nails.

4) Reamed nails tend to become infected less frequently than nonreamed nails.

 

When comparing reamed and nonreamed nails, nonreamed nails consistently perform better in areas of union, infection rates, and times to union. Previous pin tract infection predisposes to late infections, primarily following reamed nailing.Correct Answer: Nonreamed nails are capable of stimulating union more reliably than reamed nails.

 

 

3406. (2143) Q8-2568:

When inserted into intertrochanteric fractures, intramedullary hip screw devices are not associated with:

 

1) Iatrogenic fractures of the greater trochanter and lateral cortex

3) Maintenance of the shape of the upper femur in comminuted fractures

2) An improved ability to ambulate when compared to a compression hip screw

5) Less sliding

4) Increased sliding

 

Intramedullary hip screw devices do not slide as much as compression hip screws. Intramedullary hip screw devices help maintain the shape of the upper femur and less shortening and malrotation are present.Correct Answer: Increased sliding

 

 

3407. (2144) Q8-2569:

Which of the following complications is increased when intramedullary hip screws are used for intertrochanteric fractures:

 

1) Excessive sliding

3) Periprosthetic fractures

2) Excessive shortening

5) Increased cutout

4) Increased blood loss

 

Several authors have reported an increase in fractures of the femur around the distal end of an intramedullary hip screw device. Some of the fractures were due to excessively bent, ill-fitting nails; however, most fractures relate to improper locking technique. Some compression hip screw devices are also associated with periprosthetic fractures.Correct Answer: Periprosthetic fractures

 

 

 

1) Anterolateral

3) Posterolateral

2) Lateral

5) Anterior

4) Medial

 

When using indirect reduction techniques to plate the femoral diaphysis, the vastus lateralis should be elevated from the linea aspera. This exposes the posterolateral surface of the femur, which is flat and accommodates a plate well. There is minimal stripping of the blood supply with this technique.Correct Answer: Posterolateral

 

 

3409. (2146) Q8-2572:

When comparing compression hip screw to multiple cancellous screw fixation of femoral neck fractures:

 

1) There is no difference between compression hip screw and multiple cancellous screw fixation.

3) The nonunion rate associated with the compression hip screws is greater than the nonunion rate associated with multiple cancellous screws.

2) The compression hip screw is associated with superior results.

5) Only one cannulated screw is needed.

4) There is no difference in the vascularity of the femoral head following a procedure with either compression hip screws or multiple cancellous screws.

 

In prospective studies comparing multiple screws to the compression hip screw, the bulk of the compression hip screw and the additional torque applied created an increased rate of avascularity and nonunion when compared to parallel cancellous screw fixation.Correct Answer: The nonunion rate associated with the compression hip screws is greater than the nonunion rate associated with multiple cancellous screws.

 

 

3410. (2147) Q8-2573:

When performing cemented hemiarthroplasty for a displaced femoral neck fracture, polymethylmethacrylate (PMMA) should:

 

1) Always be used

3) Be substituted in an attempt to insert a press-fit prosthesis

2) Never be used

5) Not affect cardiopulmonary function

4) Be avoided in patients with cardiopulmonary disease

 

Generally, most hemiarthroplasties should be cemented to prevent the complications of osteoporotic bone fracturing, especially if the femur has bowed. The greatest contraindication to PMMA is the presence of severe cardiopulmonary disease because this can cause serious systemic complications.Correct Answer: Be avoided in patients with cardiopulmonary disease

 

 

3411. (2148) Q8-2574:

When comparing unipolar hemiarthroplasty to bipolar hemiarthroplasty for displaced femoral neck fractures:

 

1) There is increased wear with unipolar hemiarthroplasty.

3) Bipolar hemiarthroplasties are superior.

2) A dislocated bipolar hemiarthroplasty is difficult to reduce.

5) Better function is obtained with a bipolar hemiarthroplasty.

4) A bipolar hemiarthroplasty will outlast a unipolar hemiarthroplasty because of metal/polyethylene movement.

 

There is no difference in function between a unipolar or bipolar hemiarthroplasty. The disadvantages of the bipolar hemiarthroplasty include a substantial increase in cost and dislocations are difficult to reduce closed. A special circumstance is needed to justify the cost of a bipolar hemiarthroplasty.Correct Answer: A dislocated bipolar hemiarthroplasty is difficult to reduce.

 

 

 

1) Terminal branches of the medial femoral circumflex artery

3) Femoral artery

2) Ligamentum teres

5) Lateral femoral circumflex artery

4) Synovial vessels

 

The medial femoral circumflex artery provides the main blood supply to the femoral head. Trueta performed injection studies and documented that the terminal branches of the medial femoral circumflex artery provide more blood supply than any other source. The ligamentum teres provides segmental blood flow only to a portion of the femoral head.Correct Answer: Terminal branches of the medial femoral circumflex artery

 

 

3413. (2150) Q8-2576:

When is nonoperative treatment of nondisplaced femoral neck fractures indicated:

 

1) In all geriatric cases

3) In all cases

2) In all cases when osteopenia is present

5) Never indicated

4) Only when a serious contraindication to surgery exists

 

The primary treatment for nondisplaced femoral neck fractures is prophylactic open reduction internal fixation. The rate of displacement and disimpaction is significant, ranging from 15% to 30%. Complications relating to internal fixation are often less than 10%.Correct Answer: Only when a serious contraindication to surgery exists

 

 

3414. (2151) Q8-2577:

The best predictor of fracture union after a displaced femoral neck fracture is:

 

1) Quality of the bone

3) Weight bearing

2) Angle of the screws

5) Age of the patient

4) Adequacy of reduction

 

Adequacy of reduction has been shown in several studies to be the best predictor of ultimate outcome. It is not clear whether this is based on technique or the nature of the original fracture. If a fracture is more difficult to reduce, it is generally more comminuted and has sustained a higher level of trauma.Correct Answer: Adequacy of reduction

 

 

3415. (2152) Q8-2578:

The optimal number of screws to repair displaced fractures of the femoral neck is:

 

1) Three

3) Two

2) Five

5) Six

4) Four

 

Studies show that the optimal treatment for displaced fractures of the femoral neck is three parallel screws in a tripod configuration. Adding a fourth or fifth screw does not improve fixation, adds to technical difficulties, and theoretically, more vascular damage to femoral head can occur.Correct Answer: Three

 

 

 

1) Nonweight bearing

3) Crutches until union

2) Weight bearing as tolerated

5) Cane ambulation

4) Touch down gait

 

The forces acting on the hip when getting in and out of bed or on and off a commode are greater than those associated with normal ambulation. In most circumstances, a patient will apply less force to the hip when weight bearing as tolerated.Correct Answer: Weight bearing as tolerated

 

 

3417. (2154) Q8-2580:

When treating comminuted intertrochanteric fractures, a medial displacement osteotomy does not:

 

1) Place the femoral neck in valgus

3) Displace the femoral shaft medially

2) Decrease the forces on the inplant

5) Maintain normal hip anatomy

4) Create a permanent lurch

 

Medial displacement osteotomy was used prior to the routine use of the compression hip screw. To avoid cutout, a permanent lurch was accepted. The osteotomy involved medial displacement of the shaft of the femur and valgus deformity of the head and neck fragment.Correct Answer: Maintain normal hip anatomy

 

 

3418. (2155) Q8-2581:

The most sensitive modality for the diagnosis of occult fractures of the femoral neck is:

 

1) Oblique radiograph

3) Tomography

2) Computed tomography scanning

5) Radionucleide scanning

4) Magnetic resonance image scanning

 

In numerous studies, the magnetic resonance image scan has proven superior to all other modalities for the diagnosis of occult fractures of the femoral neck. In addition, it is virtually immediately positive and can be performed in an outpatient setting.Correct Answer: Magnetic resonance image scanning

 

 

3419. (2156) Q8-2582:

When an arterial injury is associated with a femoral diaphyseal fracture:

 

1) Immediate exploration of the fracture site should be performed.

3) A wide-field angiogram should be obtained to examine for lesions away from the fracture site.

2) Fluoroscopy is a safe technique to use for an angiographic procedure.

5) Fracture fixation should be performed prior to angiography.

4) Angiography is always indicated before vascular exploration.

 

Arterial injuries sometimes occur away from a fracture site. Primary exploration is not indicated unless a prolonged ischemic time mandates immediate surgery. When possible, a wide-field angiogram should be performed to visualize all associated arterial injuries.Correct Answer: A wide-field angiogram should be obtained to examine for lesions away from the fracture site.

 

 

 

1) Anterior surface of the patella

3) Interosseous wires

2) Circumferentially around the patella (cerclage wires)

5) Wires are not needed.

4) The positioning of the wire makes no difference.

 

The anterior surface of the patella is the tension surface. Control of the tension surface allows compression on the inner surface, causing the bone to absorb stress and relieve the wire of the need to support movement. This is the tension band principle.Correct Answer: Anterior surface of the patella

 

 

3421. (2158) Q8-2584:

Which of the following prognostic criteria is not associated with fractures of the tibial plateau:

 

1) Articular congruency

3) Widening of the tibial plateau

2) Stability with the knee in extension

5) Gender

4) Age of the patient

 

Articular congruency, stability with the knee in extension, and widening of the tibial plateau are important prognostic criteria. Stability with the knee in extension is an expression of not only the potential for ligamentous instability but also deformity.

Widening of a tibial plateau takes the weight-bearing surface of the tibia out from under the femur and is associated with the femur articulating an area of bone without articular cartilage.Correct Answer: Gender

 

 

3422. (2159) Q8-2585:

Angiography should be used in dislocations of the knee:

 

1) In the presence of asymmetric pulses

3) With absent pulses only

2) In all cases

5) Has been replaced by magnetic resonance image scanning

4) Does not need to be used as long as Doppler pulses are audible

 

Some controversy exists as to the role of angiography. It is clear that if pulses are not symmetric, angiography is indicated. By the time a Doppler study is needed, pulses are not palpable and angiography should be performed anyway. The need for angiography should not be determined by absent pulses, only asymmetric pulses.Correct Answer: In the presence of asymmetric pulses

 

 

3423. (2160) Q8-2586:

Most low-energy, minimally displaced tibia fractures are optimally treated with:

 

1) Fracture bracing

3) Nonreamed intramedullary nailing

2) Reamed intramedullary nailing

5) Short leg casting

4) Long leg casting

 

Fracture bracing is the classic treatment for low-energy, isolated fractures of the tibial diaphysis when a reduction can be maintained. Rapid union can be expected. Controversy exists when a reduction is necessary.Correct Answer: Fracture bracing

 

to be associated with a nonunion is:

 

1) Proximal metaphysis

3) Tibial shaft

2) Junction of the proximal metaphysis and diaphysis

5) Supramalleolar area

4) Junction of diaphysis and distal third

 

The most likely area for a tibial nonunion is the junction of the proximal diaphysis and metaphysis. When controlling for the amount of soft tissue injury, more nonunions are likely to occur in the proximal tibia because more high energy fractures occur distally.Correct Answer: Junction of the proximal metaphysis and diaphysis

 

 

3425. (2162) Q8-2588:

Which mechanism of injury is most likely associated with severe soft tissue damage from a fracture of the tibia:

 

1) Direct blow

3) Motor vehicle accident

2) Motorcycle accident

5) Low velocity gunshot

4) Sports injury

 

Direct blow fractures, particularly from a baseball bat, are far more likely to be associated with a compartment syndrome than any other mechanism. In a series by Levy and Bromberg, the incidence of compartment syndromes caused by a direct blow was approximately nine times that of other mechanisms.Correct Answer: Direct blow

 

 

3426. (2955) Q8-3461:

Patients with neglected brachial plexus injuries may achieve good functional results after shoulder joint arthrodesis if:

 

1) The function of the deltoideus muscle is preserved.

3) The function of the serratus anterior muscle is preserved.

2) The function of the pectoralis major muscle is preserved.

5) Achieving good functional results after shoulder joint arthrodesis does not depend on preserved function of shoulder girdle muscles.

4) The function of the latissimus dorsi muscle is preserved.

 

The serratus anterior muscle is the main stabilizing and motor muscle of the scapula in the anterolateral plane, which is innervated by the nervus thoracicus longus (C 5 -C 7 ). Thus, a patient who has shoulder joint arthrodesis after a neglected brachial plexus injury has an optimal level of shoulder abduction only if function of the serratus anterior muscle is preserved.Correct Answer: The function of the serratus anterior muscle is preserved.

 

 

3427. (2956) Q8-3462:

Which of the following muscles in case of its transposition has the maximum renewal of power and amplitude of elbow flexion in patients with a brachial plexus injury:

 

1) Latissimus dorsi muscle

3) Pectoralis major muscle

2) Sternocleidomastoideus muscle

5) Trapezius muscle

4) Triceps muscle

 

The latissimus dorsi muscle has a greater renewal of power and amplitude of elbow flexion than the sternocleidomastoideus muscle, the pectoralis major muscle, the triceps muscle, and the trapeziuis muscle.Correct Answer: Latissimus dorsi muscle

 

reconstructive orthopedic surgery be performed:

 

1) In normal physiological terms of reinnervation for every upper extremity muscle group

3) 3 times as long as normal physiological terms of reinnervation for every upper extremity muscle group

2) 1.5-2 times as long as normal physiological terms of reinnervation for every upper extremity muscle group

5) 6 months after the brachial plexus injury

4) 1 year after the brachial plexus injury

 

The normal speed of nerve regeneration is 1 mm per day. Nerve regeneration may be slower in patients with complications. Surgeons should not perform reconstructive orthopedic surgery until double physiological terms of nerve regeneration are over. However, surgeons should not wait longer than the double physiological term of reinnervation to perform surgery because deinnervated muscle atrophy prevents effective functional renewal after surgery.Correct Answer: 1.5-2 times as long as normal physiological terms of reinnervation for every upper extremity muscle group

 

 

3429. (2958) Q8-3464:

Which of the following kinds of trauma is the most frequent cause of brachial plexus injuries:

 

1) Automobile trauma

3) Motorcycle trauma

2) Domestic trauma

5) Industrial trauma

4) Sports trauma

 

Motorcycle trauma causes brachial plexus injuries more frequently than automobile trauma, domestic trauma, sports trauma, and industrial trauma. The high speed at which many motocyclists travel, as well as the unprotected position of motorcyclistsâ arms and shoulder girdles, cause brachial plexus injuries in motorcyclists.Correct Answer: Motorcycle trauma

 

 

3430. (2959) Q8-3465:

Which of the following branches of spinal nerve radices are damaged in patients with brachial plexus injuries:

 

1) C 5 -C 6

3) C 7 -T1

2) C 7 -C 8

5) C 8 -T4

4) C 5 -T1

The C 5 -T1 spinal nerve branches help form the brachial plexus, which innervates the upper extremity. If the brachial plexus is completely damaged, then the superior monoplegia should be examined. Patients with partial brachial plexus injuries may develop Duchenne-Erb paralysis or Klumpke's paralysis if their C 5 -C 6 and C 7 -T1 spinal nerve branches are damaged.Correct Answer: C 5 -T1

 

 

3431. (3546) Q8-4534:

The rigidity of an external fixator can be increased by:

 

1) Decreasing the distance between the bone and rods

3) Decreasing pin diameter

2) Decreasing the number of pins

5) Increasing the distance between fragment pin sets

4) Separating the half pins by less than 30º when applying anterior and anteromedial frames

 

Many factors determine the rigidity of a frame. The pin holes should be less than 30% of the bone diameter to decrease the risk of open section fracture. The bending stiffness of the pin is proportional to half the radius to the fourth power. Therefore, increasing the pin diameter to less than 30% of the bone diameter will increase the rigidity of the external fixator. Tubular rods with an 11-mm diameter are approximately twice as stiff as solid connecting rods with an 8-mm diameter. Increasing the pin spread within each bone segment to 9 cm triples the resistance to anteroposterior bending. Decreasing the bone rod distance to 2.5 cm leads to a threefold increase in resistance to transverse bending. Another way to increase stiffness of the frame is to separate half pins by greater than 45° when applying anterior and anteromedial frames together.Correct Answer: Decreasing the distance between the bone and rods

 

 

3432. (3694) Q8-7472:

Intramedullary nail fixation of an extra-articular proximal third tibial fracture is commonly associated with what malreduction deformity of the fracture site:

 

1) Apex posterior (hyperextension)

3) External rotation

2) Varus, apex anterior, and anterior displacement of the distal fragment

5) Valgus, apex anterior, and anterior displacement of the distal fragment

4) Valgus, apex anterior, and posterior displacement of the distal fragment

 

Proximal tibial fractures treated with an intramedullary nail without the use of special techniques to aid reduction are usually associated with the following deformities:

 

Valgus

 

Apex anterior

 

Posterior displacement of the distal fragment

Correct Answer: Valgus, apex anterior, and posterior displacement of the distal fragment

 

 

3433. (3695) Q8-7473:

Techniques that may be useful to avoid malreduction during intramedullary nailing of a proximal tibia fracture include all of the following except:

 

1) Nailing in a semi-extended position

3) Use of blocking screws

2) Use of a small fragment plate with unicortical screws

5) Use of a more lateral and proximal entry site

4) Use of a bump to obtain additional knee flexion

 

Flexion of the knee increases the apex anterior angulation deformity due to the pull of the patella tendon. All of the other techniques have been described to avoid malreduction during intramedullary nailing of proximal tibial fractures.Correct Answer: Use of a bump to obtain additional knee flexion

 

 

3434. (3696) Q8-7474:

To prevent apex anterior deformity during intramedullary nailing of proximal third tibial fractures, a blocking screw should be placed in what location:

 

1) Anterior to the intended ideal path of the nail

3) Medial to the intended ideal path of the nail

2) Posterior to the intended ideal path of the nail

5) Two screws should be placed obliquely to adequately block the deformity.

4) Lateral to the intended ideal path of the nail

 

To prevent apex anterior deformity, a blocking screw is placed from medial to lateral just posterior to the intended ideal posterior location of the intramedullary nail.Correct Answer: Posterior to the intended ideal path of the nail

 

 

3435. (3697) Q8-7475:

To prevent valgus deformity during intramedullary nailing of proximal third tibial fractures, a blocking screw should be placed in what location:

 

1) Anterior to the intended ideal path of the nail

3) Medial to the central axis of the tibia

2) Posterior to the intended ideal path of the nail

5) Blocking screws are not useful in preventing valgus deformity.

4) Lateral to the central axis of the tibia

 

To prevent valgus angulation, a blocking screw should be placed just lateral to the central axis of the tibia. As the nail is passed medial to the locking screw, the deformity is corrected.Correct Answer: Lateral to the central axis of the tibia

 

To prevent the less common varus deformity during intramedullary nailing of proximal third tibial fractures, a blocking screw should be placed in what location:

 

1) Anterior to the intended ideal path of the nail

3) Medial to the central axis of the tibia

2) Posterior to the intended ideal path of the nail

5) Blocking screws are not useful in preventing valgus deformity

4) Lateral to the central axis of the tibia

 

While a valgus deformity is most commonly seen when nailing a proximal third tibial fracture, correction of varus angulation can be achieved by placing a blocking screw just medial to the central axis of the tibia.

To prevent valgus angulation, a blocking screw should be placed just lateral to the central axis of the tibia. As the nail is passed medial to the locking screw, the deformity is corrected.

Correct Answer: Medial to the central axis of the tibia

 

 

3437. (3699) Q8-7477:

Which of the following statements is true concerning rotational deformities that can occur with intramedullary nailing of femoral fractures:

 

1) They are very uncommon (less than 5% incidence).

3) They rarely cause any functional complication, even when greater than 15°.

2) More than 80% are internal rotation deformities.

5) They rarely occur when the nailing is done on a fracture table.

4) Internal rotational deformities are better tolerated than external rotational deformities.

 

In a study of 76 patients who underwent intramedullary femoral nailing on a fracture table, Jaarsma and colleagues found that 28% (21 patients) had a rotational deformity of 15· or more. Twelve of these were external rotation deformities and nine were internal rotation deformities. They reported that patients with a rotational deformity had difficulties with activities like running, sports, and climbing stairs. Patients with an external rotational malalignment had more functional problems than patients with an internal rotational malalignment.

 

Correct Answer: Internal rotational deformities are better tolerated than external rotational deformities.

 

 

3438. (3700) Q8-7478:

Which of the following radiographic findings provides the best assessment of femoral rotational alignment during intramedullary nailing of a comminuted femoral shaft fracture:

 

1) Mechanical axis

3) Profile of the lesser trochanter

2) Anatomic axis

5) Outer diaphyseal diameter

4) Cortical thickness

 

While comparison of the cortical thickness and comparison of the inner and outer diaphyseal diameters are useful in assessing the accuracy of rotational alignment, comparison of the profile of the lesser trochanter between the injured and the uninjured limb likely provides the best assessment of rotational alignment, especially in a comminuted fracture. Assessment of the mechanical and anatomic axis is useful for assessing varus or valgus deformities but is not useful for assessing rotational alignment.

 

Correct Answer: Profile of the lesser trochanter

 

To assess femoral rotation using computed tomography (CT) scan, a line is drawn along the posterior border of the femoral condyles and another line drawn through the femoral neck. If the angle between these two lines is increased compared to the uninjured side, there is:

 

1) Increased external rotation of the distal fragment

3) Decreased external rotation of the distal fragment

2) Increased internal rotation of the distal fragment

4) Decreased internal rotation of the distal fragment

 

Jeanmart and colleagues described a technique for evaluating femoral malrotation using limited CT cuts through the proximal and distal femur. A line is drawn along the posterior border of the femoral condyles and another line drawn through the femoral neck. The angle between these two lines is measured and any difference between the injured and uninjured sides is compared. A decrease in angle of the fractured side indicates increased external rotation of the distal fragment, while an increase of this angle indicates an increased internal rotation of the distal femoral fragment.

 

Correct Answer: Increased internal rotation of the distal fragment

 

 

3440. (3728) Q8-7508:

The incidence of radial nerve palsy in patients sustaining a humeral shaft fracture is:

 

1) Less than 1%

3) About 10%

2) Less than 5 %

5) About 30 %

4) About 20 %

 

Correct Answer: About 10%

 

 

3441. (3729) Q8-7509:

Spontaneous recovery of a radial nerve palsy associated with a humeral shaft fracture can be expected to occur:

 

1) About 88% of the time

3) Significantly less frequently in secondary palsies

2) About 98% of the time

5) Only if the nerve is explored operatively

4) Significantly less frequently in primary palsies

 

In the literature review by Shao and investigators there was no significant difference in the recovery rate between primary (occurring at the time of injury) and secondary (occurring after the injury, or as a result of a closed reduction) nerve palsies. The recovery rate in primary nerve palsies was 88.6% and 93.1% in secondary nerve palsies.

 

Correct Answer: About 88% of the time

 

Secondary radial nerve palsies occurring as a result of fracture manipulation are:

 

1) Absolute indications for surgical exploration

3) Similar in the rate of recovery to primary nerve palsies

2) Highly uncommon (< 2% of radial nerve palsies)

5) Only seen with Holstein-Lewis fracture patterns

4) More commonly nerve lacerations than primary nerve palsies

 

Approximately 10% to 20% of nerve palsies develop during the course of treatment, commonly noted following a closed reduction, and are termed secondary nerve palsies. When a radial nerve palsy develops following fracture manipulation (secondary radial nerve palsy), many surgeons have advocated radial nerve exploration because this scenario suggests that the radial nerve might be entrapped within the fracture.

 

However, in the literature review by Shao and investigators there was no significant difference in the recovery rate between primary (occurring at the time of injury) and secondary (occurring after the injury, or as a result of a closed reduction) nerve palsies. The recovery rate in primary nerve palsies was 88.6% and 93.1% in secondary nerve palsies. The authors of this study did not state whether there was a different rate of operative treatment in these two groups.

 

Shah and Bhatti reported on 17 patients who developed secondary radial nerve palsies. Eight patients were treated with surgical exploration. Although all of the nerves were found to be intact, three were impinged on the fracture fragments and one was entrapped in scar. The other nine patients were managed nonoperatively. All patients with secondary paralysis had complete functional recovery of the radial nerve.

 

Based on the available literature, no strong evidence indicates that all secondary radial nerve palsies require immediate surgical exploration.

Correct Answer: Similar in the rate of recovery to primary nerve palsies

 

 

3443. (3731) Q8-7511:

Fracture patterns associated with the greatest incidence of radial nerve palsy are:

 

1) Comminuted fractures

3) Segmental fractures

2) Oblique fractures

5) Bilateral fractures

4) Transverse and spiral fractures

 

In a review of the literature by Shao and colleagues, transverse and spiral fracture patterns were significantly (P < .001) more likely to be associated with a radial nerve palsy than oblique or comminuted fractures

 

Fracture pattern

Incidence

# Palsies/# Total fractures

Transverse

21.2%

47/222

Spiral

19.8%

19/96

Oblique

8.4%

15/179

Comminuted

6.8%

26/382

Correct Answer: Transverse and spiral fractures

 

The classic Holstein-Lewis fracture pattern is:

 

1) A spiral fracture of the distal third of the humerus in which the radial nerve may be entrapped in the fracture

3) A spiral fracture of the middle third of the humerus in which the radial nerve may be entrapped in the fracture

2) A spiral fracture of the distal third of the humerus in which the radial nerve is commonly lacerated by the sharp fracture edge

5) A fracture pattern which is an absolute indication for radial nerve exploration when a palsy is present

4) A spiral fracture of the middle third of the humerus in which the radial nerve is commonly lacerated by the sharp fracture edge

 

In 1963 Holstein and Lewis7 reported on seven cases of radial nerve palsy that occurred in humeral shaft fractures. They stated that in their experience radial nerve palsies occurred when the fracture pattern was a spiral fracture of the distal humerus in which the distal bone fragment was displaced proximally with its proximal end deviated radialwards entrapping the radial nerve (Slide). Holstein and Lewis recommended operative treatment when a radial nerve palsy was associated with this fracture pattern and location. Holstein and Lewis recommended operative treatment when a radial nerve palsy was associated with this fracture pattern and location. While laceration of the nerve is possible, it is far less common than neuropraxia due to stretch or compression at the fracture site.

 

 

 

Slide. Diagram of Holstein-Lewis fracture pattern. The radial nerve pierces the lateral intermuscular septum as it traverses from posterior to anterior (A). Proximal and varus displacement of a spiral fracture of the distal third of the humerus may result in the radial nerve being trapped in the fracture site (B). (Image courtesy of David J. Hak, MD, MBA)

 

In the series of radial nerve palsies reported by Shah and colleagues6 , 11 patients had Holstein-Lewis fractures. Eight of these were primary palsies, while three were secondary palsies. Nine patients were treated conservatively, and two had surgical exploration of the radial nerve. In one of the patients with a secondary palsy treated operatively, the radial nerve was found to be caught in the fracture fragments. Ten patients were available for follow-up examination and all had complete recovery of radial nerve function.

 

The author agrees with Shao and researchers2 who stated that the special relationship between this fracture pattern and radial nerve palsy is not as strong as Holstein and Lewis suggested. As such, a spiral fracture pattern of the distal humerus with associated nerve palsy is not an absolute indication for radial nerve exploration.

 

Correct Answer: A spiral fracture of the distal third of the humerus in which the radial nerve may be entrapped in the fracture

 

When comparing intramedullary nailing with compression plate fixation of humeral shaft fractures, the use of intramedullary nailing is associated with an increased risk of:

 

1) Infection

3) Malunion

2) Iatrogenic nerve injury

5) Reoperation

  1. Nonunion

     

    A meta-analysis performed on the oldest three studies (Rodriguez-Merchan, Chapman and colleagues, and McCormack and researchers) indicated that plate fixation provided a risk reduction of 74% for reoperation; however, this is in large part due to the inclusion of the 19 patients who underwent removal of their retrograde flexible nails. The meta-analysis indicated that plate fixation also reduced the risk of shoulder complications, which is likely a valid finding because it included 51 antegrade interlocked intramedullary nails.

     

    A subsequent analysis of the three studies that used interlocked intramedullary nails (Chapman and colleagues, McCormack and investigators, and Changulani and researchers) also reported an increase in the risk of reoperation when intramedullary nailing was performed. This analysis found no significant difference in the percentage of patients developing nonunion, infection, or iatrogenic nerve injury.

     

    Correct Answer: Reoperation

     

     

    3446. (3735) Q8-7515:

    Approximately how frequently does iatrogenic nerve injury occur as a result of compression plate fixation in the humerus:

     

    1) 2%

    3) 10%

    2) 5%

  2. 20%

4) 15%

 

Gregory and Sanders summarized the results of several individual studies of humeral shaft fixation. Pooled data from six studies involving 302 fractures treated with compression plate fixation found no functional problems of the shoulder or elbow, 11 nonunions, 6 iatrogenic nerve palsies, and 10 infections.Correct Answer: 2%

 

 

3447. (3736) Q8-7516:

When comparing compression plate fixation with antegrade interlocked intramedullary nailing of humeral shaft fractures, the use of compression plate fixation is associated with a reduced risk of:

 

  1. Malunion

3) Shoulder pain

2) Nonunion

5) Radial nerve palsy

4) Infection

 

A meta-analysis performed on three randomized studies (Rodriguez-Merchan, Chapman and colleagues, and McCormack and researchers) indicated that plate fixation reduced the risk of shoulder complications.

A subsequent analysis of the three studies that used interlocked intramedullary nails (Chapman and colleagues, McCormack and investigators, and Changulani and researchers) found no significant difference in the percentage of patients developing nonunion, infection, or iatrogenic nerve injury.

 

Correct Answer: Shoulder pain

 

In the prospective randomized study by Chapman and colleagues that compared compression plate fixation with intramedullary nailing, the loss of elbow motion in patients treated with plate fixation was associated with:

 

1) Anterolateral surgical approach

3) Use of a broad, large fragment plate

2) Posterior surgical approach

5) Comminuted fractures

4) Fractures in the distal third of the diaphysis

 

Chapman and colleagues performed a prospective study of 89 patients with humeral shaft fractures. Patients were randomized to treatment with a locked antegrade nail or compression plate fixation. Follow-up averaged 13 months. Similar rates of healing were seen by 16 weeks, with 93% of patients healing with plate fixation and 87% of patients healing with intramedullary nailing. A significant (P = .007) incidence of shoulder pain and decreased shoulder range of motion were seen with intramedullary nailing.

Shoulder complications developed in 10 of the 32 patients (31%) treated with intramedullary nailing. In comparison, a significant (P = .03) decrease in elbow range of motion was seen in the plate fixation group, especially for fractures of the distal third of the diaphysis. Other complications were similar between the two groups. The investigators concluded that for patients requiring surgical treatment of a humeral shaft fracture, intramedullary nailing and compression plating both provide predictable methods for achieving fracture stabilization and ultimate healing.Correct Answer: Fractures in the distal third of the diaphysis

 

 

3449. (3738) Q8-7518:

When comparing antegrade interlocked intramedullary nailing with compression plate fixation of humeral shaft fractures, the use of intramedullary nailing is associated with an increased risk of:

 

1) Infection

3) Shoulder dysfunction

2) Iatrogenic nerve injury

5) Delayed union

4) Elbow stiffness

 

The rate of shoulder pain and dysfunction following antegrade interlocked humeral nails ranges from 16% to 37%. It is commonly believed that this complication is due to injury to the rotator cuff during nail insertion. To minimize complications, surgeons have recommended cutting the rotator cuff in-line with its fibers, careful retraction, and meticulous repair following nail insertion.Correct Answer: Shoulder dysfunction

 

In young patients sustaining a high-energy femoral shaft fracture, all of the following are recommended to minimize the risk of missing an ipsilateral femoral neck fracture except:

 

1) A thin-cut computed tomography (CT) scan through the hip

3) A dedicated internal rotation hip view at follow-up

2) A careful intraoperative fluoroscopic examination of the hip

5) A dedicated hip film when patients complain of hip pain

4) A dedicated external rotation hip view immediately postoperative

 

While dedicated internal rotation hip views are useful, dedicated external rotation hip views are not indicated. Because the femoral neck is anteverted, internal rotation of the hip is necessary to bring the axis of the femoral neck perpendicular to the radiographic beam.

 

Tornetta and colleagues described the use of a thin-cut computed tomography scan and dedicated anteroposterior (AP) internal rotation views of the femoral neck to decrease the incidence of missed femoral neck fractures. In addition, they recommend carefully examining the intraoperative lateral hip fluoroscopic view and obtaining dedicated AP internal rotation views of the hip at follow-up. At follow-up, patients were questioned about the presence of hip pain and, if positive, were imaged with tomography.

 

In the year prior to the institution of the protocol, seven (9%) of eighty-two patients with a femoral shaft fracture were found to have an associated femoral neck fracture. Four of those fractures were not diagnosed during the preoperative work-up or while in the operating room. Three of these four fractures were displaced when they were diagnosed and required additional surgical treatment.

 

Number of Number of Prevalence of Delayed diag femoral shaft neck and shaft fractures (%) Number

nosis of fracture

Prevalence (%)

Pre-protocol

82

7

9%

4

57%

Post-protocol

254

16

6%

1

6.3%

 

After the institution of the protocol, the incidence of missed femoral neck fractures dropped from 57% to 6.3% (Table). Table. Comparison of the Detection of Associated Femoral Neck Fractures Before and After Initiation of Protocol1

patients with fracture

patients with fractures

 

neck and shaft

femoral neck

 

Correct Answer: A dedicated external rotation hip view immediately postoperative

 

 

3451. (3791) Q8-7573:

Ipsilateral femoral neck and shaft fractures are most commonly seen in what situation:

 

1) Older patients sustaining low-energy falls

3) Young patients sustaining low-energy falls

2) Older patients sustaining high-energy injuries

5) Both young and old patients sustaining high-energy injuries

4) Young patients sustaining high-energy injuries

 

Ipsilateral femoral neck and shaft fractures typically occur in young patients (median age 34 years) sustaining high-energy injuries. It has been postulated that this injury pattern occurs as a result of an axially directed force, such as a motor vehicle accident in which the knee strikes the dashboard. The axial force causes both a fracture of the femoral neck and a comminuted fracture of the femoral diaphysis. Due to the mechanism of injury, associated knee injuries are commonly seen, including patella fractures, contusions, and lacerations. In a meta-analysis of 659 patients, knee injuries were present in half of the patients.

Because these frequently result from high-energy injuries, associated multisystem injuries are seen in 73% to 100% of patients.

The shaft fracture is commonly mid-diaphyseal and frequently comminuted, as is often seen in high-energy fractures. The femoral neck fracture is commonly vertical, basilar, and minimally or nondisplaced.

Correct Answer: Young patients sustaining high-energy injuries

 

The pattern of femoral neck fractures typically seen in association with ipsilateral femoral shaft fractures are:

 

1) Widely displaced and easily diagnosed

3) Nondisplaced, vertical, basicervical

2) Nondisplaced, transverse, transcervical

5) Non comminuted, subcapital

4) Severely comminuted, subcapital

 

The femoral neck fracture is commonly vertical, basilar, and minimally or nondisplaced. The shaft fracture is commonly mid-diaphyseal and frequently comminuted, as is often seen in high-energy fractures.

Correct Answer: Nondisplaced, vertical, basicervical

 

 

3453. (3793) Q8-7575:

Femoral neck nonunion is most commonly seen following which treatment method for an ipsilateral femoral neck and shaft fracture:

 

1) Reconstruction nailing

3) Lag screw fixation of femoral neck and plate fixation of shaft

2) Lag screw fixation of femoral neck and retrograde nailing of shaft

5) Dynamic hip screw fixation of femoral neck and plate fixation of shaft

4) Dynamic hip screw fixation of femoral neck and retrograde nailing of shaft

 

Femoral neck nonunions are more commonly seen with reconstruction nailing. Watson and Moed caution against the use of a reconstruction nail for fixation of femoral neck and shaft fractures. In their report of complications following treatment of ipsilateral femoral neck and shaft fractures, they had eight femoral neck nonunions, and six (75%) of these occurred in patients treated with a reconstruction nail. They point out that the implant was designed to provide adjunctive fixation into the head and neck to decrease the moment arm when stabilizing proximal femoral shaft fractures, not for the fixation of ipsilateral femoral neck fractures. The screws inserted through the nail into the femoral head and neck were not designed to function as compression lag screws, and they have poor sliding characteristics due to the short working length of the screw within the nail. Because of the limited sliding capacity, loading may results in impaction of the femoral head cancellous bone and potentially screw cut-out.

Alternatively, if the femoral head and neck are securely anchored in strong cancellous bone, then resorption of bone at the fracture site effectively leads to a progressively increased distance between the fracture edges. Equally problematic is the inability to place screws optimally in the femoral head and neck, because the position of the screws is fixed by the location of the proximal nail holes.

 

 

Dynamic hip screw stabilization can be effectively used instead of lag screw fixation for basicervical femoral neck fractures. Correct Answer: Reconstruction nailing

A âmissedâ ipsilateral femoral neck fracture is most commonly due to:

 

1) Iatrogenic injury at the time of antegrade nailing

3) Failure to review patients abdominal computed tomography (CT) scan

2) Iatrogenic injury at the time of retrograde nailing

5) Malpractice, because a missed injury should never occur if standard protocols are followed

4) Inadequate preoperative imaging of the femoral neck

 

While iatrogenic injury can occur with a malpositioned entry site for an antegrade nail, most ipsilateral femoral neck fractures are due to the injury. Initial radiographs are often of poor quality, and the fracture may not be adequately visualized. While reviewing a patientâs abdominal CT may identify an occult femoral neck fracture, the cuts are often 1 cm width and may not adequately image the femoral neck. A thin-cut CT scan of the hip is recommended. Even in the presence of standard protocols, ipsilateral femoral neck fractures can still be missed.

 

Tornetta and colleagues described the use of a thin-cut CT scan and dedicated anteroposterior (AP) internal rotation views of the femoral neck to decrease the incidence of missed femoral neck fractures. In addition, they recommend carefully examining the intraoperative lateral hip fluoroscopic view and obtaining dedicated AP internal rotation views of the hip at follow-up. At follow-up, patients were questioned about the presence of hip pain and, if positive, were imaged with tomography.

 

In the year prior to the institution of the protocol, seven (9%) of eighty-two patients with a femoral shaft fracture were found to have an associated femoral neck fracture. Four of those fractures were not diagnosed during the preoperative work-up or while in the operating room. Three of these four fractures were displaced when they were diagnosed and required additional surgical treatment.

 

 

After the institution of the protocol, the incidence of missed femoral neck fractures dropped from 57% to 6.3%. Correct Answer: Inadequate preoperative imaging of the femoral neck

 

3455. (3891) Q8-7771:

Which of the following is an advantage of lateral positioning over supine positioning when performing antegrade intramedullary nailing of a subtrochanteric femur fracture with an intact lesser trochanter:

 

1) Provides improved pulmonary ventilation

3) Allows faster setup and positioning

2) Eliminates valgus sag at fracture site

5) Provides more accurate rotational alignment

4) Provides easier alignment of the distal segment to the flexed proximal segment

 

Subtrochanteric fractures can pose challenges in reduction due to the muscle attachments proximal and distal to the fragment. The gluteus medius and gluteus minimus attach to the greater trochanter and abduct the proximal fragment. The iliopsoas attaches to the lesser trochanter, flexing and externally rotating the proximal fragment. In the lateral position, it may be easier to align the distal fragment with the flexed proximal fragment. In the supine position, it may be necessary to place a Schantz pin in the proximal fragment to counteract the deforming forces acting on the proximal fragment.

 

Pulmonary ventilation is better in the supine position. In the lateral position, you must be careful to avoid valgus sag at the fracture site. Careful attention to correct rotational alignment is necessary in both the supine and lateral position.

Correct Answer: Provides easier alignment of the distal segment to the flexed proximal segment

 

In the Russell-Taylor classification of subtrochanteric femur fractures, a fracture involving the piriformis fossa without comminution of the lesser trochanter is classified as:

 

1) IA

3) IIA

2) IB

5) IIIA

4) IIB

 

In the Russell-Taylor classification system, a fracture involving the piriformis fossa without comminution of the lesser trochanter is classified as a type IIA.

The subtrochanteric classification system proposed by Russell and Taylor focused on the integrity of the piriformis fossa and thus the practicality of using a piriformis entry intramedullary nail for fixation of the fracture. Type I fractures do not have extension into the piriformis fossa, while type II fractures involve extension into the piriformis fossa.

 

Each of these categories is subcategorized into two groups. In type IA, the lesser trochanter is intact, and these patterns can be fixed with a standard antegrade intramedullary nail. In type IB patterns, there is a fracture involving the lesser trochanter. A standard antegrade nail is not feasible in this pattern, as the proximal locking screw cannot achieve purchase in the lesser trochanter. Treatment of this pattern generally requires a cephalomedullary nail that has a screw, or screws, which are positioned in the femoral neck and head. Alternatively, plating with a blade plate, 95º dynamic condylar screw or a locking proximal femur plate may be used.

 

The type II fractures are also subdivided into two categories. In type IIA patterns, there is no significant comminution of the lesser trochanter. While in type IIB patterns, there is comminution of the greater trochanter along with loss of the medial femoral cortex, including the lesser trochanter. Because of the involvement of the piriformis, most surgeons have avoided the use of a piriformis entry nail in these two patterns of fractures and traditionally have relied on some from of plate stabilization.

 

Table.Russell-Taylor classification of subtrochanteric fractures.

 Â

 Â

Lesser trochanter

Intact

Comminuted

 

Piriformis fossa

Intact

IA

IB

Involved in fracture

IIA

IIB

 

 

 

Slide 1 Russell-Taylor subtrochanteric fracture classification system. In type I fractures the piriformis fossa is intact, while type II fractures are characterized by fracture extension into the piriformis fossa. In subcategory A, the lesser trochanter remains intact, while in subcategory B, the lesser trochanter is a separate comminuted fragment. (Image courtesy of David J. Hak, MD, MBA.)

Correct Answer: IIA

 

The deformity commonly seen in a subtrochanteric femur fracture where the lesser trochanter is intact includes:

 

1) Adduction and extension of the proximal fragment

3) Abduction and extension of the proximal fragment

2) Adduction and flexion of the proximal fragment

5) Predominantly internal rotation of the proximal fragment

4) Abduction and flexion of the proximal fragment

 

The most commonly seen deformity in subtrochanteric femur fractures is abduction and flexion of the proximal fragment.

Subtrochanteric fractures can pose challenges in reduction due to the muscle attachments proximal and distal to the fragment. The gluteus medius and gluteus minimus attach to the greater trochanter and abduct the proximal fragment. The iliopsoas attaches to the lesser trochanter, flexing and externally rotating the proximal fragment. The short external rotators (piriformis, superior and inferior gemellus) and the obturator internus also cause external rotation of the proximal fragment. The adductors attaching distally result in varus and shortening.

 

Correct Answer: Abduction and flexion of the proximal fragment

 

 

3458. (3894) Q8-7774:

The strength of the implant used for fixation of subtrochanteric femur fractures is critical because the subtrochanteric femur has:

 

1) High tensile forces medially and high compressive forces laterally

3) High torsional loads

2) High compressive forces medially and high tensile forces laterally

5) High bending loads in simple fracture patterns

4) Low compressive forces medially and high tensile forces laterally

 

The subtrochanteric femur is a region of high stress, placing high demands on implants used for fixation. There are high compressive forces medially, and high tensile forces laterally. When there is medial comminution, the implant used for fixation is subjected to high bending load.

 

Subtrochanteric fractures are generally slower to heal than intertrochanteric fractures. This slow rate of healing places additional demands on the implants.

Correct Answer: High compressive forces medially and high tensile forces laterally

 

 

3459. (3895) Q8-7775:

Which of the following implants is least desirable for fixation of a comminuted subtrochanteric femur fracture:

 

1) Blade plate

3) Trochanteric entry cephalomedullary nail

2) Piriformis entry cephalomedullary nail

5) Locking proximal femur plate

4) 135º dynamic/sliding hip screw

 

The 135º dynamic or sliding hip screw was designed for use in intertrochanteric hip fractures. Its use in subtrochanteric fractures can be successful, but additional periosteal disruption is necessary for reduction and fixation may lead to healing problems.

Additionally, the proximal fragment may rotate about the single proximal compression screw.

The use of the 135º dynamic or sliding hip screw is specifically problematic in reverse oblique fracture patterns. In a retrospective study of reverse oblique fractures, loss of fixation was seen in 56% (9 of 16) treated with a 135º dynamic or sliding hip screw, compared with 13% (2 of 16) treated with a 95º blade plate.

 

Correct Answer: 135º dynamic/sliding hip screw

 

When the ankle is dorsiflexed, the fibula moves in:

 

1) Internal rotation and is translated laterally

3) External rotation and is translated laterally

2) Internal rotation and is translated proximally

5) External rotation and is translated proximally

4) External rotation and is translated distally

 

The anterior portion of the talus is wider than the posterior portion, such that when the ankle goes from plantarflexion to dorsiflexion the mortise widens by 1.5 mm. During ankle dorsiflexion, the fibula translates proximally, is externally rotated, and translates laterally. With plantarflexion, the fibula is pulled distally, internally rotated, and medially translated.

 

Correct Answer: External rotation and is translated proximally

 

 

3461. (3897) Q8-7777:

Structures that make up the syndesmosis include all of the following EXCEPT:

 

1) Anterior inferior tibiofibular ligament (AITFL)

3) Tibiofibular interosseous ligament

2) Anterior talofibular ligament (ATFL)

5) Inferior transverse tibiofibular ligament

4) Posterior inferior tibiofibular ligament (PITFL)

 

The syndesmosis between the tibia and fibula at the ankle consists of five defined structures (Slide):

  1. Anterior inferior tibiofibular ligament (AITFL)

  2. Posterior inferior tibiofibular ligament (PITFL)

  3. Inferior transverse tibiofibular ligament

  4. Tibiofibular interosseous membrane

  5. Tibiofibular interosseous ligament

 

 

Slide. Ligamentous structures comprising the ankle syndesmosis. IOM = interosseous membrane, IOL

= interosseous ligament, AITFL = anterior inferior tibiofibular ligament, PITFL = posterior inferior tibiofibular ligament, ITL = inferior transverse tibiofibular ligament. (Image courtesy of David J. Hak, MD, MBA)

Slide

 

 

 

Correct Answer: Anterior talofibular ligament (ATFL)

 

The recommended orientation for syndesmotic screw fixation with respect to the coronal plane of the tibia is:

 

1) Angled 15° posterior

3) Parallel to the posterior tibial surface

2) Angled 30° posterior

5) Angled 30° anterior

4) Angled 15° anterior

 

While variation exists in the morphology of the syndesmosis, most researchers recommend that the orientation of syndesmotic fixation be angled 25° to 30° anterior to the coronal plane (Slide). The fibula normally sits in the incisura, which is oriented posterolaterally.

 

Slide Because of the location of the syndesmosis, most researchers recommend that the orientation of syndesmotic fixation be angled 25° to 30° anterior to the coronal plane. (Image courtesy of David J. Hak, MD, MBA)

Slide 6

 

 

 

Correct Answer: Angled 30° anterior

 

Laboratory and clinical studies clearly demonstrate that the optimal syndesmotic fixation is achieved with which of the following constructs:

 

1) A single 3.5-mm cortical screw placed through 3 cortices

3) Two 3.5-mm cortical screws placed through 3 cortices

2) A single 4.5-mm cortical screw placed through 4 cortices

5) None of the above have been shown to be clearly superior

4) Two 4.5-mm cortical screws placed through 4 cortices

 

Controversy exists concerning the optimal syndesmotic fixation. Laboratory and clinical studies have failed to clearly indicate that one fixation technique is superior to another.

There is conflicting data concerning whether a larger diameter screw is superior. Using tricortical fixation, one biomechanical study found no difference between using a 4.5-mm and a 3.5-mm diameter screw. However, in another biomechanical study using quadricortical fixation, the 4.5-mm diameter screw had significantly higher resistance to shear stress during weightbearing.

 

Disagreement exists whether the syndesmotic screw should engage three or four cortices. Proponents of engaging four cortices claim that if the screw should break it can still be removed by grabbing the far screw tip. Proponents of engaging only three cortices claim that screw failure is less common, because the screw can move (windshield wiper) in the metaphyseal bone.

 

Investigators performed a prospective randomized study comparing three vs four cortices of syndesmotic screw fixation in 64 patients. The investigators routinely removed the quadricortical screws at 2 months but only removed the tricortical screws if they were symptomatic. Two tricortical screws that loosened were removed. The patients Olerud and Molander functional scores were significantly better in the tricortical group at 3 months (77 points vs 66 points, P=.025), but at 1 year there was no significant difference (92.6 points vs 85.7 points, P=.017).

 

In another surgeon randomized study comparing three vs four cortices of fixation, investigators found no difference in the rate of hardware breakage or reduction loss.

Correct Answer: None of the above have been shown to be clearly superior

 

 

3464. (3900) Q8-7780:

Which of the following static radiographic measurements suggests syndesmotic disruption:

 

1) Tibiofibular overlap of 50% on anteroposterior (AP) view

3) Tibiofibular overlap of 7 mm on AP view

2) Tibiofibular overlap of 2 mm on mortise view

5) Tibiofibular clear space of 5 mm on mortise view

4) Tibiofibular clear space of 7 mm on AP view

 

Slide 1

 

 

Tibiofibular overlap measures the overlap of the two bones at a distance 1 cm proximal to the ankle joint (Slide 1). On the AP view, the tibiofibular overlap should be greater than 6 mm or greater than 42% of the width of the fibula, while on the mortise view , the tibiofibular overlap should be greater than 1 mm.9

 

 

Slide 1. The tibiofibular overlap measures the overlap of the two bones at a distance 1 cm proximal to the ankle joint. On the AP view, the tibiofibular overlap should be greater than 6 mm or greater than 42% of the

 

width of the fibula, while on the mortise view , the tibiofibular overlap should be greater than 1

mm. (Image courtesy of David J. Hak, MD, MBA)

 

The tibiofibular clear space is defined as the distance between the medial border of the fibula and the lateral border of the incisura (Slide 2). Measured 1 cm proximal to the ankle joint, the tibiofibular clear space should be greater than 6 mm on the AP and mortise views.9

 

Slide 2. The tibiofibular clear space is defined as the distance between the medial border of the fibula and the lateral border of the incisura.

Measured 1 cm proximal to the ankle joint, the tibiofibular clear space should be less than 6 mm on the AP and mortise views. (Image courtesy of David J. Hak, MD, MBA)

Slide 2

 

 

 

Correct Answer: Tibiofibular clear space of 7 mm on AP view

 

 

3465. (3981) Q8-8233:

The difference between a Pipkin type I fracture and Pipkin type II fracture is:

 

1) Size of the femoral head fracture fragment

3) Location of the femoral head fracture

2) Degree of comminution

5) Direction of the hip dislocation

4) Presence of associated femoral neck fracture

 

The difference between a Pipkin type I fracture and a Pipkin type II fracture is the location of the femoral head fracture. A Pipkin type I fracture is infrafoveal, whereas a Pipkin type II fracture is suprafoveal. A femoral head fracture with an associated femoral neck fracture is classified as a Pipkin type III fracture. The Brumback classification differentiates based on the direction of the hip dislocation, whereas the simpler Pipkin classification does not include this information.

 

Correct Answer: Location of the femoral head fracture

 

 

 

1) A Pipkin type I fracture with 3 mm of displacement

3) A Pipkin type III fracture

2) A Pipkin type II fracture with 3 mm of displacement

5) A Pipkin type I fracture with concentric reduction and a stable hip

4) A comminuted Pipkin type I fracture with nonconcentric reduction

 

Following closed reduction of the hip, nonoperative treatment can be considered for Pipkin type I and type II fractures if the following criteria are met:

  1. Anatomic or near anatomic reduction with < 2 mm displacement

  2. A stable hip

  3. No interposed fragments preventing a congruent hip reduction

While many Pipkin type I fractures can be treated nonoperatively, it is unusual to be able to successfully treat Pipkin type II fractures nonoperatively.

 

Pipkin type III fractures, which require operative treatment, are femoral head fractures associated with a femoral neck fracture. Correct Answer: A Pipkin type I fracture with concentric reduction and a stable hip

 

3467. (3983) Q8-8235:

Recommended implants for internal fixation of femoral head fractures include all of the following except:

 

1) Countersunk 3.5-mm screws

3) Herbert screws (Zimmer Corp., Warsaw, IN)

2) Countersunk cannulated 3.5-mm screws and washers

5) Bioabsorbable implants

4) Acutrak screws (Acumed LLC, Hillsboro, OR)

 

Fracture fixation can be accomplished with standard 3.5-mm or 2.7-mm countersunk lag screws, self-compressing screws (Herbert screws), or bioabsorbable pins.9 The use of 3.5-mm cannulated screws and washers is not advised because investigators have reported dissociation of the washer from the screw head with production of third body degenerative changes of the hip.

 

Correct Answer: Countersunk cannulated 3.5-mm screws and washers

 

 

3468. (3984) Q8-8236:

A femoral head fracture associated with a femoral neck fracture is classified as a:

 

1) Pipkin type I fracture

3) Pipkin type III fracture

2) Pipkin type II fracture

5) Pipkin type V fracture

4) Pipkin type IV fracture

 

A femoral head fracture associated with a femoral neck fracture is classified as a Pipkin type III fracture. A Pipkin type I fracture refers to an infrafoveal femoral head fracture. A Pipkin type II fracture refers to a suprafoveal femoral head fracture. A Pipkin type IV fracture refers to a femoral head fracture associated with an acetabular fracture. While the Brumback classification includes a type V, the simpler Pipkin classification only includes types I to IV.

 

Correct Answer: Pipkin type III fracture

 

 

 

1) Higher rate of avascular necrosis

3) Higher rate of sciatic nerve palsy

2) Higher rate of heterotopic ossification

5) Greater difficulty inserting lag screws

4) Poorer functional outcome

 

In a study by Swiontkowksi and colleagues, the use of the anterior approach was associated with a significantly higher incidence of heterotopic ossification. The anterior approach avoids additional compromise to the femoral head blood supply, and allows easier access to the femoral head fracture fragment, which is usually anteromedial.

 

The use of the Kocher-Langenbeck posterior approach is reported to have a 3.2 times higher incidence of subsequent avascular necrosis then the Smith-Petersen approach. However, in one study the use of the anterior approach was associated with a significantly higher incidence of heterotopic ossification. The anterior approach avoids additional compromise to the femoral head blood supply and allows easier access to the femoral head fracture fragment, which is usually anteromedial.

 

The rate of traumatic sciatic nerve palsy has been reported from 10% to 23%. Iatrogenic sciatic nerve palsy can occur due to traction during a posterior approach, but it would be uncommon to occur following an anterior approach.

Correct Answer: Higher rate of heterotopic ossification

 

 

3470. (3987) Q8-8239:

The highest rate of associated nerve injury in sacral fractures occurs with:

 

1) Denis zone 1, which is a fracture lateral to the foramen

3) Denis zone 2, which is a fracture through the foramen

2) Denis zone 1, which is a fracture medial to the foramen

5) Denis zone 3, which is a fracture medial to the foramen

  1. Denis zone 3, which is a fracture lateral to the foramen

     

    In a series of 236 patients, Denis and researchers reported the highest rate of associated nerve injuries occurred in fractures that were medial to the foramen and extending into the spinal canal. This location is classified as a Denis zone 3 fracture. The lowest rate of associated nerve injury was found with Denis zone 1 fractures, which are lateral to the foramen. Denis zone 2 fractures, which occur through the foramen, had an intermediate incidence of nerve injury.

     

    Correct Answer: Denis zone 3, which is a fracture medial to the foramen

     

     

    3471. (3988) Q8-8240:

    Associated neurologic injury is present in what percentage of sacral fractures:

     

    1) 5%

    3) 25%

    2) 15%

  2. 75%

4) 45%

 

Sacral fractures occur in approximately 45% of pelvic fractures; an associated neurologic injury of the lumbosacral plexus may occur in 25% of sacral fractures.

Correct Answer: 25%

 

 

 

  1. An iliosacral screw into the S1 vertebral body

3) Tension band plate fixation

2) Iliosacral screws in S1 and S2

5) An iliosacral screw that traverses the S1 vertebral body and achieves purchase in the opposite iliac wing

4) Triangular osteosynthesis

 

The term triangular osteosynthesis describes fixation constructs that involve placement of pedicle screws in the lower lumbar and the posterior ilium in conjunction with iliosacral screws. In a biomechanical study, triangular osteosynthesis was shown to be superior to iliosacral screws alone for fixation of unstable transforaminal sacral fractures.

 

Correct Answer: Triangular osteosynthesis

 

 

 

1) Denis zone 1 fracture

3) Denis zone 3 fracture

2) Denis zone 2 fracture

5) Roy-Camille type 1 fracture

4) I-shaped fracture

 

This fracture is classified as a Denis zone 1 pattern. The Denis classification divides the sacrum into three zones (Slide 1): Zone 1: Fractures are located lateral to the neural foramina.

Zone 2: Fractures pass through the foramina.

Zone 3: Fractures are located medial to the foramen and involve the spinal canal.

While transverse sacral fractures have been described based on the pattern morphology as H, U, lambda, and T, a simple vertical fracture would not usually be classified by the letter âIâ morphology.

Roy-Camille and colleagues, and Strange-Vognsen and Lebech further classified transverse Denis zone 3 fractures. In this classification, type 1 fractures show only kyphotic angulation at the fracture. Type 2 fractures have kyphosis and partial anterior translation. Type 3 fractures have kyphosis along with complete translation. Type 4 fractures have segmental comminution of the S1 body due to axial compression (Slide 2).

 

Slide 1: The Denis classification of sacral fractures is illustrated. (Image courtesy of David J. Hak, MD, MBA)

 

 

 

Slide 2: The descriptive patterns of transverse sacral fractures are illustrated: A = H pattern, B = U pattern, C = lambda pattern, and D = T pattern. (Image courtesy of David J. Hak, MD, MBA)

 

 

Correct Answer: Denis zone 1 fracture

 

 

 

1) Test great toe dorsiflexion

3) Test sensation in the 1st web space of the foot

2) Test ankle toe plantarflexion

5) Test perianal sensation

4) Test sensation on the medial border of the foot

 

Associated neurologic injury, especially of the lower sacral plexus, may not be appreciated unless a thorough examination is performed. Injuries to the S2 to S5 nerve roots can be overlooked because they do not supply motor or sensory supply to the lower leg. The S2 nerve innervates the musculature that forms the external urethral and anal sphincters. S4 and S5 nerves provide sensation to the penis, labia, urethra, posterior scrotum, and anal canal. The bladder and rectum are innervated primarily by the pelvic splanchnic autonomic nerves from S2 to S4. Continence and sexual function require at least unilateral preservation of the S2 and S3 nerve roots.

 

Clinical examination of patients sustaining sacral fractures requires more than routine examination of lower extremity sensory and motor function. Additional examination is required to identify injuries to the lower sacral plexus. A rectal examination should be performed to evaluate sphincter contraction. Light touch and pinprick sensation must be assessed for the perianal dermatomes of S2 to S5. In addition to the perianal wink, the bulbocavernosus and cremasteric reflexes must also be assessed.

 

Extensor hallicus longus (great toe dorsiflexion) is innervated by the deep peroneal nerve. Flexor hallicus longus (great toe plantarflexion) is innervated by the tibial nerve. The medial border of the foot is innervated by the L4 dermatome, and the lateral border of the foot is innervated by the S1 dermatome. The 1st web space is innervated by the L5 dermatome (deep peroneal nerve).

 

Correct Answer: Test perianal sensation

 

 

3475. (4) Q9-32:

The diameter of the nidus in most osteoid osteomas is:

 

1) 1 mm-2 mm

3) 15 mm-30 mm

2) 5 mm-10 mm

5) 50 mm-75 mm

4) 30 mm-50 mm

 

Osteoid osteomas, by strict definition, have a limited growth potential. The nidus is typically 5 mm-10 mm and is never over 1.5 cm. The nidus is lytic in appearance and often has an area of central ossification. There may be large amounts of reactive bone formation that surround the nidus.

 

Correct Answer: 5 mm-10 mm

 

 

 

1) Location in the vertebral body is common.

3) The femoral neck is the single most common site.

2) Location in flat bones is common.

5) Craniofacial involvement is common.

4) In long bones, the lesions are most likely in the mid-diaphysis.

 

Osteoid osteomas have some specific locations to remember: The ends of long bones

 

Femoral neck (most common location) Vertebra

 

 

Usually in lamina or base of the pedicle Virtually never in the vertebral body

Highly unlikely locations

 

 

Flat bones Craniofacial bones

Correct Answer: The femoral neck is the single most common site.

 

 

3477. (6) Q9-34:

Which of the following describes the presentation of a patient with an osteoid osteoma:

 

1) Pain with weight bearing

3) Pain relieved by aspirin and nonsteroidal anti-inflammatory drugs

2) Hard mass but no pain

5) Morning stiffness

4) Intermittent discomfort relieved by rest

 

Osteoid osteomas have one of the most characteristic presentations of all bone tumors. The pain is often completely relieved with aspirin and nonsteroidal anti-inflammatory drugs. Parents of affected children have stated that their child will finish a whole bottle of aspirin in 1 week. The pain commonly occurs at night and will wake the child if he or she does not take some form of medication.

 

Correct Answer: Pain relieved by aspirin and nonsteroidal anti-inflammatory drugs

 

 

 

1) Beneath the anterior longitudinal ligament

3) In the spinous process

2) In the vertebral body

5) In the transverse process

4) In the lamina or base of the pedicle

 

When osteoid osteomas occur in the spine, they are most commonly located in the lamina or the base of the pedicle. Generally, they do not occur in the vertebral bodies, spinous processes, or transverse processes.

Common location

 

In the lamina or base of the pedicle Uncommon locations

 

 

 

Vertebral body Spinous process Transverse process

Correct Answer: In the lamina or base of the pedicle

 

 

3479. (8) Q9-36:

Which of the following describes the radiographic features of an osteoid osteoma:

 

1) Ovoid, compact, and heavily mineralized intramedullary lesion with thorny spicules

3) Surface lesion with spiculated bone formation

2) Large, nodular, and heavily mineralized lesion on the surface of the bone

5) Cortically based lytic metaphyseal lesion with a sclerotic border

4) Well-demarcated nidus surrounded by a distinct zone of sclerosis

 

 

Osteoid osteomas have a common radiographic appearance: 5 mm-10 mm lytic nidus surrounded by sclerosis

The other possible answers describe the radiographic features of other lesions:

 

 

 

Bone island: Ovoid, compact, and heavily mineralized intramedullary lesion with thorny spicules Parosteal osteosarcoma: Large, nodular, heavily mineralized lesion on the surface of the bone Periosteal osteosarcoma: Surface lesion with spiculated bone formation

 

 

Non-ossifying fibroma: Cortically based lytic metaphyseal lesion with a sclerotic border Correct Answer: Well-demarcated nidus surrounded by a distinct zone of sclerosis

Which of the following describes the microscopic features of osteoid osteoma:

 

1) Interlacing network of bone trabeculae in a loose, vascular, and stromal connective tissue

3) Scattered giant cells in a mononuclear cell background with chicken-wire calcification

2) Compact cortical bone

5) Prominent mature bone formation and low-grade malignant spindle cell tumor

4) Pleomorphic spindle cells with osteoid production

 

Osteoid osteomas have a characteristic histologic appearance. The nidus has an interlacing network of bone trabeculae in a loose, vascular, and stromal connective tissue. There is densely sclerotic bone surrounding the lesion.

The other possible answers describe specific lesions:

 

Bone island: Compact cortical bone with radiating spicules

 

 

Chondroblastoma: Scattered giant cells in a mononuclear cell background with chicken-wire calcification Osteosarcoma: Pleomorphic spindle cells with osteoid production

 

 

Parosteal osteosarcoma: Prominent mature bone formation and low-grade malignant spindle cell tumor Correct Answer: Interlacing network of bone trabeculae in a loose, vascular, and stromal connective tissue

 

3481. (10) Q9-38:

Which of the following statements is true concerning osteoid osteomas of the hands and feet:

 

1) Swelling, but no pain

3) Lack of surrounding sclerosis about the nidus

2) Marked swelling and synovitis of the adjacent joints

5) Lesions in the feet are twice as common as those in the hand

4) Localization in the phalanges is rare

 

Osteoid osteomas of the hand and feet can be diagnostic challenges. Prominent new bone formation may be present that makes it difficult to locate the nidus. In contrast, lesions that border articular surfaces may show no reactive sclerosis. CT scans will often show the nidus. There may be marked swelling in superficial lesions and synovitis of adjacent joints.

 

Important points to remember include:

 

The most common location is the phalanges.

 

Hand lesions are twice as common as foot lesions.

Correct Answer: Marked swelling and synovitis of the adjacent joints

 

Which of the following tumors has the best prognosis (least risk of pulmonary metastases):

 

1) Parosteal osteosarcoma

3) Classic (or ordinary) osteosarcoma

2) Periosteal osteosarcoma

5) Osteosarcoma following irradiation

4) Osteosarcoma in Pagetâs disease

 

Parosteal osteosarcoma is a low-grade form of osteosarcoma. The tumor is well differentiated and has a very low metastatic rate. When local control of the lesion is achieved, the risk of pulmonary metastases is only about 3 to 5%. If the tumor contains areas of high-grade sarcoma (such as high-grade malignant fibrous histiocytoma, fibrosarcoma, or osteosarcoma), the lesion is called a de-differentiated parosteal osteosarcoma. De-differentiated parosteal osteosarcoma has a more ominous prognosis with a high rate of metastases.

 

The risk of pulmonary metastases for all of the possible answers are as follows:

 

Tumor Approximate Risk

Periosteal osteosarcoma 20%-30% Classic (or ordinary) osteosarcoma 80%-90% Osteosarcoma in Pagetâs disease 80%-90% Osteosarcoma following irradiation 80%-90% Correct Answer: Parosteal osteosarcoma

 

 

3483. (18) Q9-48:

Which of the following tumors has the best prognosis (least risk of pulmonary metastases):

 

1) Rhabdomyosarcoma

3) Malignant fibrous histiocytoma

2) Primitive neuroectodermal tumor

5) Pleomorphic liposarcoma

4) Myxoid liposarcoma

 

All of the lesions listed in the possible answers are high-grade soft tissue sarcomas, except for myxoid liposarcoma, which is an intermediate-grade sarcoma. Generally, low-, intermediate-, and high-grade sarcomas have the following approximate risk of the patient developing pulmonary metastases:

 

Grade

Approx. Risk

Tumor

Low (Grade 1)

5%-10%

Grade 1 liposarcoma, Grade malignant fibrous histiocytoma

Intermediate (Grade 2)

20%-30%

Myxoid liposarcoma

High (Grade 3,4)

50%+

Rhabdomyosarcoma, primitive neuroectodermal tumor, malignant fibrous histiocytoma, pleomorphic liposarcoma

Correct Answer: Myxoid liposarcoma

 

 

 

 

 

Slide 1 Slide 2 Slide 3 Slide 4

 

 

 

Slide 5

A 17-year-old boy has a 4-month history of severe pain in his right distal tibia. The pain is especially prominent at night. He has difficulty sleeping and concentrating at school. The lateral radiograph of his right distal tibia and clinical photograph is shown in Figure 1. A computerized tomogram (CT) and axial T1 weighted magnetic resonance imaging (MRI) scan are shown in Figures 2 and 3. The most likely diagnosis is:

 

1) Osteosarcoma

3) Ewing's tumor

2) Osteomyelitis

5) Stress fracture

4) Osteod osteoma

 

Osteoid osteomas are common tumors in young individuals. Patients usually present with intense pain and they can often localize the exact point of discomfort.

CT findings are often quite characteristic (Figure 4):

 

 

small nidus 5 mm to 10 mm in the cortex or in a sub-periosteal location often there is central ossification in the nidus

 

sclerosis surrounding the nidus

On MRI, it is often more difficult to see the nidus than on CT scanning. On T1 weighted images the nidus can often be seen as small round lesion (Figure 5). In the cortex, because the nidus often has mineralization which is low signal (as is the cortex which the nidus is sitting in), one may have trouble detecting the nidus.

 

The differential diagnosis of osteod osteomas include:

  1. Stress fracture

  2. Osteomyelitis

 

Because of the characteristic imaging findings, one can make this diagnosis based upon the CT or MRI findings. Correct Answer: Osteod osteoma

 

 

 

 

Slide 1 Slide 2 Slide 3 Slide 4

A 35-year-old man has a 9-month history of a soft tissue mass in his forearm. The mass has been steadily enlarging over time. The Tl and T2 weighted axial magnetic resonance imaging (MRI) scans are shown in Figures 1 and 2. A biopsy was performed and is shown in Figure 3. The most likely diagnosis is:

 

1) Malignant fibrous histiocytoma

3) Lipoma

2) Epitheloid sarcoma

5) Synovial sarcoma

4) Desmoid tumor

 

The forearm mass shown is low signal on TI weighted images and high signal on T2 weighted images. These signal characteristics are non-specific and may represent either a sarcoma or a benign lesion.

The histologic section shows a biphasic lesion with epithelial and spindle cells (Figure 4). This is a typical characteristic of synovial sarcoma.

The histologic appearance of the other lesions is:

malignant fibrous histiocytoma â storiform pattern with pleomorphic spindles cells and histiocytes epitheloid sarcoma â nodular growth pattern with central necrosis and epithelial quality of the cells lipoma â mature adipose cells

 

desmoid - mature spindle cells with abundant collagen and no cellular atypia Correct Answer: Synovial sarcoma

Which of the following cartilage tumors has the worst prognosis (highest risk of pulmonary metastases):

 

1) Chondrosarcoma arising in Ollierâs disease

3) Dedifferentiated chondrosarcoma

2) Chondrosarcoma arising in Maffucciâs syndrome

5) Grade 1 intramedullary chondrosarcoma

4) Grade 2 intramedullary chondrosarcoma

 

All of the possible answers (except one) are either low- or intermediate-grade bone sarcomas. Dedifferentiated chondrosarcoma is a high-grade spindle cell sarcoma. In dedifferentiated chondrosarcomas, there is a low-grade chondrosarcoma (grade 1 or 2) which is intimately admixed with a high-grade spindle sarcoma (grade 3 or 4 anaplastic spindle cell sarcoma -osteosarcoma, malignant fibrous histiocytoma, or fibrosarcoma).

 

 

The risk of pulmonary metastases in the lesions for all of the possible answers are as follows:

 

Tumor

 

Approximate Risk

 

 

 

Chondrosarcoma arising in Ollier's disease

 

1% to 5%

 

 

 

Chondrosarcoma arising in Maffucci's syndrome

 

1% to 5%

 

 

 

Grade 2 intramedullary chondrosarcoma

 

20% to 30%

 

 

 

Grade 1 intramedullary chondrosarcoma

 

1% to 5%

 

 

Grade

 

Risk

 

 

 

Low (Grade 1)

 

5% to 10%

 

 

 

Intermediate (Grade 2)

 

20% to 30%

 

 

 

High (Grade 3,4)

 

50%+

 

Generally, low-, intermediate-, and high-grade sarcomas have the following risk of systemic metastases:

 

 

Correct Answer: Dedifferentiated chondrosarcoma 3487. (23) Q9-59:

Synovial chondromatosis occurs most commonly in which of the following joints:

 

1) Hip

3) Shoulder

2) Knee

5) Ankle

4) Elbow

 

Synovial chondromatosis most commonly occurs in the knee. Other joints where it may occur include the hip, ankle, foot, and shoulder.

Correct Answer: Knee

 

To which of the following organs do high-grade intramedullary osteosarcomas most commonly metastasize:

 

1) Brain

3) Adrenals

2) Lungs

5) Kidneys

4) Other bones

 

Osteosarcomas most commonly spread to the lungs. Plain chest radiographs may not reveal small lesions. Computerized tomography of the chest is the most sensitive method to detect nodules that are 3 mm to 5 mm in diameter. Other sites of metastases include other bones and visceral organs.

 

Correct Answer: Lungs

 

 

3489. (31) Q9-67:

Which of the following benign bone tumors may occasionally (approximately 2% of the time) metastasize to the lungs:

 

1) Chondromyxoid fibroma

3) Giant cell tumor

2) Osteochondroma

5) Osteofibrous dysplasia

4) Fibrous dysplasia

 

Giant cell tumor of bone is considered a benign tumor. Occasionally, this tumor can metastasize to the lungs. The incidence in different series has varied between 1% and 9%. Most large series show an incidence of 1% to 2%.

When a giant cell tumor metastasizes to the lungs, the biologic behavior of the lesions can be variable. The overall prognosis is favorable, with 75% to 85% surviving rate. The treatment of the pulmonary metastases depends on the clinical scenario:

 

One to five nodules

Most centers would recommend thoracotomy and removal.

 

Greater than 10 nodules

With large numbers of nodules, there are basically two options -chemotherapy or observation. In some patients, with observation alone, the nodules regress.

Generally, in approximately 20% of patients, the disease will progress with either enlargement of the pulmonary disease or spread to other organs.

Correct Answer: Giant cell tumor

 

 

3490. (32) Q9-69:

Which of the following groups of soft tissue sarcomas commonly metastasize to lymph nodes:

 

1) Liposarcoma and malignant fibrous histiocytoma

3) Malignant fibrous histiocytoma and clear cell sarcoma

2) Liposarcoma and clear cell sarcoma

5) Malignant fibrous histiocytoma and hemangioendothelioma

4) Synovial sarcoma and epithelioid sarcoma

 

The four main soft tissue sarcomas that may metastasize to lymph nodes are epithelioid sarcoma, synovial sarcoma, rhabdomyosarcoma and clear cell sarcoma. It is uncommon for the other lesions to metastasize to lymph nodes.

Correct Answer: Synovial sarcoma and epithelioid sarcoma

 

 

 

1) Giving way with pivoting

3) Recurrent non-traumatic effusions

2) Pain on ascending and descending steps

5) Medial joint line pain

4) Movie sign (pain following prolonged sitting)

 

Pigmented villonodular synovitis is a synovial proliferative disorder characterized by atraumatic recurrent effusions and cystic erosions of the periarticular bone surfaces. Patients commonly have multiple knee aspirations with blood-tinged fluid. The effusions are often large.

 

Correct Answer: Recurrent non-traumatic effusions

 

 

3492. (36) Q9-73:

Which of the following best describes the current treatment of osteosarcoma in adolescents:

 

1) Wide resection alone

3) Wide resection, multi-agent chemotherapy, and radiation

2) Wide resection and multi-agent chemotherapy

5) Interferon and radiation

4) Wide resection and radiation

 

Osteosarcoma is a high-grade malignant bone tumor. Historical studies have shown that with surgery alone, 80% to 90% of patients develop pulmonary metastases. Multi-agent chemotherapy has dramatically improved long-term survival from 10% to 20% to 60% to 70%.

 

Currently, the chemotherapy is given for 8 to 12 weeks preoperatively. The tumor is resected and patients are given a 6-month course of maintenance chemotherapy.

Correct Answer: Wide resection and multi-agent chemotherapy

 

 

3493. (37) Q9-74:

Which of the following best describes the current treatment of a Grade 2 chondrosarcoma of the distal femur:

 

1) Wide resection alone

3) Wide resection, multi-agent chemotherapy, and radiation

2) Wide resection and multi-agent chemotherapy

5) Interferon and radiation

4) Wide resection and radiation

 

Chondrosarcomas are treated by surgical resection alone. Chemotherapy and irradiation play a limited role in treatment. Generally, chondrosarcoma grow slowly, therefore irradiation or chemotherapy will have little effect on the tumor cells.

Correct Answer: Wide resection alone

 

 

 

1) Wide resection alone

3) Wide resection, multi-agent chemotherapy, and radiation

2) Wide resection and multi-agent chemotherapy

5) Interferon and radiation

4) Wide resection and radiation

 

Parosteal osteosarcomas are treated by surgical resection alone. Chemotherapy and irradiation play a limited role in treatment. Parosteal osteosarcomas are composed of two populations of cells â a low-grade spindle cell sarcoma (fibrosarcoma, Grade 1 or 2) and well differentiated osteoblasts that produce large amounts of mature and organized bone. Generally, these cells grow slowly, therefore irradiation or chemotherapy will have little effect on the tumor cells.

 

In contrast, if there is a dedifferentiated component (a high-grade osteosarcoma, fibrosarcoma, or malignant fibrous histiocytoma), the prognosis is poor with a high risk of pulmonary metastases. In this scenario, the patient would be treated with chemotherapy. There would still be no role for irradiation.

 

Correct Answer: Wide resection alone

 

 

3495. (39) Q9-76:

In which of the following tumors is the bone scan the least sensitive:

 

1) Metastatic breast cancer

3) Osteosarcoma

2) Metastatic lung cancer

5) Osteoid osteoma

4) Multiple myeloma

 

The technetium bone scan is sensitive in osteosarcoma and osteoid osteoma because the tumor cells produce osteoid. The technetium isotope localizes well to the sites of osteoid production. In breast cancer, the bone scan is also sensitive as there is often significant amounts of new bone formation. In lung cancer, there is prominent bone destruction. With this bone destruction, there is also some attempt at bone formation and the technetium will localize at these sites. If there is prominent bone destruction with no bone formation, then the bone scan will be negative (false-negative).

 

With multiple myeloma, there is an uncoupling of the normal relationship between bone resorption and bone formation. In myeloma, the plasma cells secrete a factor that activates and recruits osteoclasts to resorb bone. There is seldom a compensatory formation of bone. This lack of coupling between bone resorption and bone formation leads to the high percentage of false-negatives in patients with multiple myeloma.

 

Correct Answer: Multiple myeloma

 

 

3496. (40) Q9-77:

In patients with destructive lesions secondary to metastatic breast cancer, the cells responsible for the bone destruction are the:

 

1) Breast cancer cells

3) Osteoclasts

2) Fibroblasts

5) Langerhansâ cells

4) Histiocytes

 

Interestingly, in metastatic bone disease and multiple myeloma, it is not the tumor cells that destroy the bone, but the osteoclasts. The tumor cells secrete a factor that recruits and activates osteoclasts. These osteoclasts then resorb the bone, and the tumor cells grow and fill the holes in the bone.

 

Correct Answer: Osteoclasts

 

 

 

1) Eosinophil

3) Fibroblast

2) Osteoclast

5) Vasoformative cells

4) Langerhansâ cell

 

Eosinophilic granuloma has been traditionally considered as part of a spectrum of histiocytic disorders called histiocytosis X. This spectrum includes eosinophilic granuloma, Hand-Schüller-Christian Disease and Letterer-Siwe Disease. In recent years, these disorders have been grouped under the single term Langerhans cell granulomatosis. The proliferating cell is not a histiocyte, but rather the dendritic cells, which are named Langerhans' cells. Langerhans cell granulomatosis can be divided into three principal types:

 

 

 

Single bone involvement (no visceral involvement) Multiple bone involvement (no visceral involvement)

 

Multiple bone involvement and visceral disease (lung, liver, spleen, lymph nodes, and skin)

Histologically, one may see many eosinophils; however, the proliferating cell is the Langerhans' cell. The Langerhans' cells are characterized by:

 

 

 

A folded or grooved nucleus A sharp nuclear membrane Eosinophilic cytoplasm

 

Correct Answer: Langerhansâ cell

 

 

3498. (42) Q9-79:

Which of the following carcinomas has the highest propensity to metastasize to the hand:

 

1) Prostate

3) Lung

2) Kidney

5) Breast

4) Thyroid

 

Most carcinomas metastasize to the spine, ribs, pelvis, skull, and proximal limb girdles. It is uncommon for metastases to occur in the hands or feet. A notable exception is lung carcinomas. Lung carcinomas metastasize to the distal skeleton (acral metastases). Over 50% of patients with distal (acral metastases) will have a lung carcinoma as the primary tumor.

 

Correct Answer: Lung

 

 

3499. (43) Q9-80:

What is the most common site of bone metastases:

 

1) Proximal femur

3) Pelvis

2) Proximal humerus

5) Hands/feet

4) Spine

 

Bone metastases most commonly occur in the axial skeleton, with the vertebral bodies as the most common site. The common distribution is to:

 

 

 

 

Spine Ribs Pelvis Skull

 

Proximal long bones

Lesions in the hands and feet are uncommon. When these distal metastases do occur, approximately one-half will be secondary to lung cancers.

Correct Answer: Spine

 

 

3500. (44) Q9-81:

Hypercalcemia commonly occurs in which of the following bone lesions:

 

1) Osteosarcoma

3) Malignant fibrous histiocytoma

2) Multiple myeloma

5) Hemangioendothelioma

4) Chondrosarcoma

 

Serum hypercalcemia is a common problem in cancer patients. The most common cancers in which this complication may occur include:

 

 

Multiple myeloma Lymphoma

 

 

Metastatic lung cancer Metastatic breast cancer

The symptoms of hypercalcemia include:

 

Acute Chronic

 

Anorexia Visual changes

 

Constipation Coma

 

Polyuria/polydipsia Nausea/vomiting Lethargy

Inability to concentrate Need for more sleep

The treatment of hypercalcemia is threefold:

 

 

Hydration Saline diuresis

 

 

Diphosphonates: Intravenous pamidronate can correct the hypercalcemia within 48 hours Correct Answer: Multiple myeloma

 

3501. (45) Q9-82:

Which of the following lesions often involves multiple bones in the same extremity in young patients:

 

1) Osteosarcoma

3) Chondroblastoma

2) Giant cell tumor

5) Chondromyxoid fibroma

4) Hemangioendothelioma

 

Most bone tumors are solitary lesions with a low incidence of multicentricity. Vascular tumors are an exception. These tumors may involve multiple bones. Patients with hemangioendothelioma present with multiple lytic lesions. These lytic lesions often occur in the same extremity in young patients.

 

Correct Answer: Hemangioendothelioma

 

 

 

1) Serum calcium

3) Serum erythrocyte sedimentation rate

2) Serum protein electrophoresis

5) Serum alkaline phosphatase

4) Serum phosphate

 

Serum alkaline phosphatase levels will be abnormal in approximately 96% of patients. The elevations vary from mild to profound. Despite the extensive bone turnover, the serum calcium levels are normal. If the serum calcium level is elevated, one should look for other conditions such as primary hyperparathyroidism.

 

The serum calcium level and erythrocyte sedimentation rate may be elevated in patients with multiple myeloma. The serum protein electrophoresis will show a monoclonal spike in patients with multiple myeloma.

Correct Answer: Serum alkaline phosphatase

 

 

3503. (47) Q9-84:

In evaluating the radiographs of a patient with Pagetâs disease, which of the following would be suggestive of malignant change:

 

1) Enlargement of the bone

3) Marked bowing

2) Coarsened trabeculae

5) Cortical bone destruction

4) Multiple lucent areas

 

Pagetâs disease is a remodeling disease in which the normal cortices are resorbed and replaced with remodeled cortices. In this condition, there are coarsened trabeculae, lucent areas, and bone enlargement. Bowing may occur and varies from mild to marked.

 

Cortical bone destruction and the presence of a soft tissue mass are common findings in patients where Pagetâs disease has undergone malignant change. Both MRI and CT scans are excellent modalities to detect malignant change. As the remodeling occurs, there should not be any destruction of the cortical bone.

 

Correct Answer: Cortical bone destruction

 

 

 

1) Chondroblastoma

3) Non-ossifying fibroma

2) Enchondroma

5) Giant cell tumor

4) Osteoid osteoma

 

Of all the benign tumors listed in the possible answers, giant cell tumors are the most prone to recur locally. Historical studies have shown local failure rates as high as 30% to 40% following curettage and grafting. The most modern technique employs curettage and reconstruction with methyl methacrylate. The local failure rate with this technique is 5% to 20%. Some centers also use adjuvants such as phenol, liquid nitrogen, and hydrogen peroxide.

 

The other lesions have low rates of recurrance:

 

 

Chondroblastoma 5%-10%

 

 

Enchondroma <5%

 

 

Non-ossifying fibroma <5%

 

 

 

Osteoid osteoma <5% Correct Answer: Giant cell tumor

3505. (49) Q9-86:

Which of the following descriptions would apply to the radiographic findings in a patient with adamantinoma:

 

1) Multiple poorly demarcated lytic lesions (>5)

3) Eccentric, cortically based lytic lesion with a well-formed sclerotic rim

2) Eccentric, lytic, expansile lesion in the proximal tibial metaphysis

5) Central medullary mineralized lesion with rings and stipples; no cortical abnormalities

4) Multiple lucencies in the tibial diaphysis with sclerosis of the intervening bone

 

Adamantinoma is a peculiar and rare bone tumor that almost exclusively involves the tibia. In some cases, when adamantinoma occurs in the tibia and fibula, there are multicentric lesions. If one finds multiple bone involvement (sites other than the tibia and fibula), the diagnosis of adamantinoma is unlikely.

 

The radiographic appearance is quite characteristic. The most common appearance is mixed sclerotic and lytic lesions. Often, one will find multiple lucencies with intervening sclerosis of the bone. Seventy percent of the lesions involve the mid-shaft of the tibia, with the remaining lesions involving the proximal or distal tibia.

 

The other possible answers describe other bone lesions:

 

Aneurysmal bone cyst:

 

Non-ossifying fibroma:

 

An eccentric, lytic, expansile lesion in the proximal tibial metaphysis.

 

An eccentric, cortically based lytic lesion with a well-formed sclerotic rim.

 

Enchondroma: A central medullary mineralized lesion with rings and stipples that has no cortical abnormalities.

Correct Answer: Multiple lucencies in the tibial diaphysis with sclerosis of the intervening bone

 

 

 

1) A mixed pattern of inflammatory cells, edema, and new blood vessels

3) Spindle cells and immature bone trabeculae

2) Langerhans' cells and eosinophils

5) Uniform population of small blue cells

4) Epithelial cells in a fibrous background

 

There are several different histologic patterns for adamantinoma: basaloid, spindle, tubular, squamoid, and osteofibrous-dysplasia like. Each of these patterns shares the basic finding of epithelial cells in a fibrous background.

 

The other possible answers describe various bone lesions:

 

Osteomyelitis: A mixed pattern of inflammatory cells, edema, and new blood vessels Eosinophilic granuloma: Langerhans' cells and eosinophils

Fibrous dysplasia: Spindle cells and immature bone trabeculae

 

Ewing's tumor: Uniform population of small blue cells Correct Answer: Epithelial cells in a fibrous background

 

 

3507. (52) Q9-89:

The most common early symptom of a sacrococcygeal chordoma is:

 

1) Pelvic pain

3) Sacral nerve dysfunction

2) Obstipation

5) Easy fatigability

4) Urinary hesitancy

 

Pelvic pain is the most common early symptom in patients with chordomas. As the mass grows anteriorly and through the sacral foramina, a constellation of symptoms may occur. Late findings may include:

 

 

Obstipation secondary to obstruction of the rectum Sacral nerve dysfunction

Correct Answer: Pelvic pain

 

 

3508. (54) Q9-91:

Which of the following describes the histologic appearance of a chordoma:

 

1) Uniform population of small blue cells

3) Sheets of plasma cells

2) Pleomorphic spindles cells that directly produce bone

5) A mixture of inflammatory cells, new blood vessels, and edema

4) Cords or nest of cells with a vacuolated cytoplasm in a myxoid intercellular matrix

 

Chordomas have a characteristic appearance:

 

Cords or nests of solid cells that have a vacuolated cytoplasm embedded in a myxoid intercellular matrix

 

The other possible answers describe specific lesions:

 

Ewing's tumor: Uniform population of small blue cells Osteosarcoma: Pleomorphic spindle cells that produce bone Myeloma: Sheets of plasma cells

Osteomyelitis: A combination of mixed inflammatory cells, edema, and new blood vessels Correct Answer: Cords or nest of cells with a vacuolated cytoplasm in a myxoid intercellular matrix

 

 

3509. (55) Q9-92:

The most common radiographic appearance of a chordoma is:

 

1) Marked sclerosis

3) Purely lytic with scattered calcific densities

2) Mixed lytic-sclerosis

5) Vertebral collapse with flattening and widening

4) Marked cortical thickening with coarsened trabeculae

 

Chordomas most commonly have a purely lytic appearance. Occasionally, they may be purely sclerotic or have a mixed appearance of lytic-sclerosis.

In the sacrococcygeal area, chordomas are typically purely lytic. The tumor grows anteriorly and expands centripetally with a periosteal layer. The soft tissue mass often has calcific densities scattered throughout. These calcifications are usually mild to moderate.

 

Correct Answer: Purely lytic with scattered calcific densities

 

 

3510. (56) Q9-93:

Which of the following describes the histologic features of conventional chordoma:

 

1) Physaliferous cells and high-grade pleomorphic spindle cells

3) Uniform population of small blue cells

2) Lobular growth pattern and physaliferous cells

5) Sheets of cells with eccentric nucleus and peripheral nuclear chromatin pattern

4) Organoid pattern of epithelial cells in a fibrous background

 

 

The principal histologic features of conventional chordomas are: Lobular growth pattern

 

 

Nests of solid cells with a vacuolated cytoplasm Physaliferous cells

Dedifferentiated chordomas have the typical features: lobular growth pattern, physaliferous cells, and a myxoid background. The other answers describe specific tumors:

 

Uniform population of small blue cells â Ewings tumor

 

 

Organized pattern of epithelial cells in a fibrous background â Metatastic carcinoma Sheets of cells with eccentric nucleus, peripheral chromatin pattern â Myeloma

Correct Answer: Lobular growth pattern and physaliferous cells

 

A 15-year-old male has an eccentric, destructive lesion in the distal femoral metaphysis that is purely lytic and extends into the soft tissue. The cortex overlying the lesion has been destroyed, but there is a thin rim of periosteal bone that is continuous.

Based on the radiographic findings, the most likely diagnosis is:

 

1) Chondroblastoma

3) Osteoid osteoma

2) Osteochondroma

5) Chondrosarcoma

4) Aneurysmal bone cyst

 

The radiographic features of aneurysmal bone cyst are:

 

 

Expansile, lytic lesion that destroys the cortex Purely lytic

 

Continuous rim of periosteal bone over the lesion The radiographic features of the other lesions are:

Chondroblastoma: Lytic lesion in the epiphysis

Sclerotic rim surrounding the lesion

One-third of the radiographs have matrix mineralization

 

Osteochondroma: Surface lesion with cortical sharing

Medullary cavity of the osteochondroma is continuous with the host bone Metaphyseal based

Point away from the joint

 

Osteoid osteoma: Small lytic nidus (5 mm-10 mm)

Nidus is surrounded by sclerotic bone

 

Chondrosarcoma: Large lesions with cortical changes: Cortical thickening

Large cortical erosions Cortical breakthrough

Correct Answer: Aneurysmal bone cyst

 

 

3512. (61) Q9-98:

All of the following statements are true concerning osteoblastomas except:

 

1) They may grow to large size

3) Surrounding sclerosis may be minimal or absent

2) There is often a lack of nocturnal pain that is responsive to aspirin

5) Histologically, osteoblastoma is similar to osteoid osteoma

4) The lesion is usually 5 mm to 10 mm in diameter

 

In contrast to osteoid osteomas, which have a limited growth potential (5 mm to 10 mm, occasionally 10 mm to 15 mm), osteoblastomas may grow to large size. When they occur in the spine, osteoblastomas may press on the spinal cord and nerve roots causing neurologic symptoms and even deficits.

 

Histologically, osteoblastomas are similar to osteoid osteomas. Osteoblastomas differ from osteoid osteomas in that:

 

 

There is often a lack of nocturnal pain that is responsive to aspirin Surrounding sclerosis may be minimal or absent

Correct Answer: The lesion is usually 5 mm to 10 mm in diameter

 

Forty percent of osteoblastomas involve the axial skeleton. Which of the following statements is true:

 

1) The vertebral body is the most common location.

3) The presence of aneurysmal bone cyst changes within the lesion is rare.

2) The posterior elements are the most common site.

5) Patients are generally over 50 years of age.

4) Neurologic symptoms are unusual.

 

Osteoblastomas commonly occur in the spine. The lesions involve the posterior elements, may cause neurologic symptoms, and commonly occur in young patients.

 

The following statements are true concerning osteoblastomas of the spine: Posterior elements are the most common site.

 

Neurologic symptoms are common.

 

Expansion of the posterior elements is common.

 

 

Aneurysmal bone cyst-like changes are commonly seen histologically. The patients are generally young.

Correct Answer: The posterior elements are the most common site.

 

 

 

 

 

Slide 1 Slide 2 Slide 3 Slide 4

 

 

 

Slide 5 Slide 6

A 17-year-old man has a 4-month history of severe pain in his right distal tibia. The pain is especially prominent at night. He has difficulty sleeping and concentrating at school. The lateral radiograph of his right distal tibia and clinical photograph is shown in Figure 1. A CT and axial T1-weighted MRI scan are shown in Figures 2 and 3. Which of the following historical findings would help in establishing the diagnosis:

 

1) Pain that occurs only with activity

3) Morning stiffness

2) Pain that occurs after running 2 to 3 miles

5) Pain on the sole of his foot associated with his first step in the morning

4) Complete resolution of the pain with aspirin or ibuprofen

 

Patients with osteoid osteomas have a characteristic pain pattern. Often, osteoid osteoma pain is completely relieved by aspirin or nonsteroidal anti-inflammatory drugs (NSAIDs). Children may finish an entire bottle of ibuprofen in just 1 week in order to avoid the pain.

 

The other possible answers describe common symptoms of other musculoskeletal conditions: Stress fractures: Pain only with activity

Exertional compartment syndrome: Morning stiffness; inflammatory arthritis Ankylosing spondylitis or rheumatoid arthritis: Pain that occurs after running 2 to 3 miles

Planter fascitis: Pain with the first step in the morning on the sole of the foot Correct Answer: Complete resolution of the pain with aspirin or ibuprofen

 

 

 

 

 

Slide 1 Slide 2 Slide 3

A 17-year-old man has a 4-month history of severe pain in his right distal tibia. The pain is especially prominent at night. He has difficulty sleeping and concentrating at school. The lateral radiograph of his right distal tibia is shown in Figure 1. A CT and axial T1-weighted MRI scan are shown in Figures 2 and 3. Which of the following treatment options should be recommended to the patient and family:

 

1) Open or needle biopsy, preoperative chemotherapy, and wide resection

3) Below knee amputation

2) Open or needle biopsy, preoperative chemotherapy, and external beam irradiation

5) Nonsteroidal anti-inflammatory medications

4) Long leg cast for 4 weeks, followed by a patellar tendon bearing cast

 

In this case, the diagnosis is osteoid osteoma. Osteoid osteomas are common tumors in young individuals. Patients usually present with intense pain, and they can often localize the exact point of discomfort.

CT findings are often characteristic:

 

 

A small nidus, 5 mm to 10 mm, in the cortex or in a sub-periosteal location Central ossification is often present in the nidus

 

Sclerosis is found surrounding the nidus

The pain of osteoid osteomas is often dramatically relieved by aspirin or nonsteroidal anti-inflammatory drugs (NSAIDs). Kniesel and Simon reported on a series of patients treated with NSAIDs in the Journal of Bone and Joint Surgery. Approximately 50% of the patients had long-term relief of their pain.

 

Correct Answer: Nonsteroidal anti-inflammatory medications

 

 

 

 

 

Figure 1 Figure 1 Figure 2 Figure 2

 

 

 

Figure 3 Figure 3

A 17-year-old man has a 4-month history of severe pain in his right distal tibia. The pain is especially prominent at night. He has difficulty sleeping and concentrating at school. The lateral radiograph of his right distal tibia is shown in Figure 1. A CT and axial T1-weighted MRI scan are shown in Figures 2 and 3. The stage of this tumor according to the system of the Musculoskeletal Tumor Society:

 

1) Stage 1

3) Stage 3

2) Stage 2

5) Stage III

4) Stage II

 

 

From the information provided in the question: The lesion is symptomatic.

 

The CT and MRI scan are diagnostic of an osteoid osteoma.

Although osteoid osteomas may cause severe pain, they do not grow and destroy bone structure. Accordingly, this lesion is an active benign lesion - Stage 2.

The Surgical Staging System of the Musculoskeletal Tumor Society is a useful system to both predict prognosis and plan treatment. The system for benign lesions is divided into three groups: inactive (latent), active, and aggressive.

 

Inactive (latent) Stage 1

 

Active Stage 2

 

 

Aggressive Stage 3 Correct Answer: Stage 2

Refers to lesions that are not causing pain and show no evidence of active growth. Stage 1 lesions are generally treated with observation only.

 

Refers to lesions which are causing pain or some form of disability. If a lesion has weakened the structure of the bone such that fracture may occur, the lesion would also be considered a Stage 2 lesion.

 

Refers to lesions which are large, have broken into the soft tissues, or have caused a pathologic fracture. These lesions are usually prone to local recurrence and have the potential of causing a major problem for the patient.

 

 

 

 

Figure 1 Figure 2 Figure 3

A 35-year-old man has a 9-month history of a soft tissue mass in his forearm. The mass has been steadily enlarging over time. The T1- and T2-weighted axial MRI scans are shown in Figures 1 and 2, respectively. A biopsy was performed and is shown in Figure 3. The most appropriate treatment of this mass would be:

 

1) Observation

3) Preoperative radiation therapy and wide resection

2) Simple excision

5) External beam irradiation and chemotherapy

4) External beam irradiation alone

 

The radiographic appearance of the lesion, low signal on T1-weighted images and high signal on T2-weighted images, is nonspecific. Virtually all sarcomas are low signal on T1-weighted images and high signal on T2-weighted images. However, one must remember that not all lesions that are low on T1-weighted images and high on T2-weighted images are malignant.

 

The histologic section shows a bimorphic tumor with both spindle cells and epithelial cells. This is the classic appearance of synovial sarcoma. This lesion is a high-grade malignancy and would be treated with preoperative irradiation and wide resection. Other acceptable answers are:

 

 

Wide resection and postoperative radiation

 

Preoperative chemotherapy, radiation therapy, and wide resection

High-grade sarcomas cannot be cured without surgery, hence the possible answers listed that do not include surgery are wrong. Simple excision would result in prompt local recurrence as the margins would be positive. Observation is a poor choice, as the tumor would continue to grow and probably metastasize.

 

Correct Answer: Preoperative radiation therapy and wide resection

 

 

3518. (78) Q9-122:

Which of the following variables is a significant risk factor for distant metastases in patients with soft tissue sarcomas:

 

1) Age

3) Size

2) Sex

5) Presence of pain

4) Location

 

Size and grade are probably the 2 most significant factors that predict the risk of metastases. A recent study reported the correlation between size and distant metastases:

 

Size Risk of distant metastases

 

1 cm-5 cm

20%

6 cm-10 cm

41%

11 cm-15 cm

53%

Correct Answer: Size

 

Which of the following soft tissue sarcomas most commonly occurs in children younger than 10 years old:

 

1) Malignant fibrous histiocytoma

3) Synovial sarcoma

2) Fibrosarcoma

5) Epithelioid sarcoma

4) Rhabdomyosarcoma

 

Rhabdomyosarcoma is the most common soft tissue sarcoma in children younger than 10 years old. The most malignant subtype of rhabdomyosarcoma, alveolar rhabdomyosarcoma, occurs in the trunk and extremities.

Correct Answer: Rhabdomyosarcoma

 

 

3520. (80) Q9-124:

Which of the following soft tissue sarcomas is the most responsive to chemotherapy:

 

1) Malignant fibrous histiocytoma

3) Synovial sarcoma

2) Fibrosarcoma

5) Epithelioid sarcoma

4) Rhabdomyosarcoma

 

Rhabdomyosarcoma is the most common soft tissue sarcoma in children. Rhabdomyosarcoma is very sensitive to both chemotherapy and radiation. Long-term survival in children who present with localized disease is 60% to 70%.

Correct Answer: Rhabdomyosarcoma

 

 

3521. (81) Q9-125:

Which of the following agents decreases the rate of skeletal complications and delays the time of the first skeletal complication in patients with metastatic breast cancer:

 

1) Calcitonin

3) Aredia (pamidronate disodium, Novartis)

2) Taxol (paclitaxel, Bristol-Myers Squibb)

5) Zofran (ondansetron, Glaxo Wellcome)

4) Adriamycin (adrenocorticotropic hormone, Pharmacia & Upjohn)

 

Pamidronate is a disphosphonate that is highly effective in stopping osteoclastic bone resorption in patients with metastatic breast cancer. A randomized study showed that intravenous pamidronate (90 mg monthly for 12 months) lowered the rate of skeletal complications and delayed the median time for the first skeletal complication from 7.0 to 13.1 months. Virtually all patients with bone metastases and breast cancer now receive intravenous pamidronate.

 

 

 

 

Calcitonin â used as an anti-resorptive agent in patients with Pagets disease Taxol â used as an anti-estrogen agent in patients with breast cancer Adriamycin â used as a cytotoxic chemotherapy agent

 

Zofran â used as an anti-nausea agent

Correct Answer: Aredia (pamidronate disodium, Novartis)

 

Which of the following cell types is directly responsible for the resorption of bone in patients with metastatic breast carcinoma:

 

1) Breast carcinoma cells

3) Osteoblasts

2) Osteocytes

5) C cells of the parathyroid glands

4) Osteoclasts

 

Bone resorption in metastatic breast cancer is directly caused by osteoclasts. The tumor cells secrete a factor that activates the osteoclasts, which then resorb the bone mineral.

 

Factors implicated in tumor mediated bone resorption include: Parathyroid related protein (PTHrP)

 

Interleukins 1 and 6

 

 

Tumor necrosis factor alpha Correct Answer: Osteoclasts

 

3523. (83) Q9-127:

Which of the following groups of carcinomas have the highest risk for metastases to bone:

 

1) Lung, breast, kidney, and prostate

3) Lung, breast, gastric, and cervix

2) Lung, gastric, rectal, and sweat gland

5) Breast, ovarian, prostate, and esophagus

4) Lung, prostate, cervix, and ovarian

 

There are approximately 1.4 million new cancer cases in the United States each year. Of these cases, the most common carcinomas to metastasize to bone are: lung, breast, prostate, and kidney.

 

Virtually any malignancy can metastasize to bone, but not as commonly as the 'big four': Lung

 

 

 

Breast Prostate Kidney

 

Correct Answer: Lung, breast, kidney, and prostate

 

 

3524. (84) Q9-128:

Allografts are commonly used to reconstruct large segment defects following the resection of tumors. Non-union of the allograft-host junction occurs in approximately what percentage of patients:

 

1) 1% to 2%

3) 15% to 35%

2) 5% to 10%

5) 75% to 90%

4) 50% to 60%

 

Allografts and custom prostheses are the 2 major methods to reconstruct defects following resection of bone tumors. The major complications following allograft reconstruction include: infection, fracture, breakdown of the articular cartilage, and non-union.

Non-union of the allograft host junction occurs in about 17% to 33% of patients. The non-union risk is greater when chemotherapy or radiation is needed. Diaphysis to diaphysis junctions are the unions that are the most difficult to unite.

Correct Answer: 15% to 35%

 

Allografts are commonly used to reconstruct large segment defects. The rate of deep infection following allograft reconstruction is:

 

1) 1% to 2%

3) 10% to 15%

2) 2% to 5%

5) 50% to 75%

4) 25% to 50%

 

Allografts and custom prostheses are the 2 major methods to reconstruct defects following resection of bone tumors. The major complications following allograft reconstruction include: infection, fracture, breakdown of the articular cartilage, and non-union.

 

Infection is the most devastating complication and occurs in about 10% to 15% of patients. Correct Answer: 10% to 15%

 

3526. (86) Q9-130:

Which of the following agents is used to protect against hemorrhagic cystitis that occurs following chemotherapeutic treatment with ifosfamide or cyclophosphamide:

 

1) Leucovorin

3) 2-mercaptoethanesulfonate (MESNA)

2) Granulocyte colony-stimulating factor

5) Amifostine (Ethoyl)

4) Ondansetron (Zofran)

 

Hemorrhagic cystitis is a severe complication associated with ifosfamide and cyclophosphamide treatment. Both of these agents are used in regimens for osteosarcoma. These agents produce an acrolein metabolite that causes acute sterile hemorrhagic cystitis. 2-mercaptoethanesulfonate (MESNA) neutralizes this compound and prevents the cystitis.

 

The other possible answers are also used to ameliorate the side effects of chemotherapy:

 

Leucovorin (folic acid): Rescue cells following high dose methotrexate (folic acid analog

that does not require reduction by the enzyme dihydrofolate reductase, bypasses effects of methotrexate)

 

Granulocyte-colony stimulating factor:

 

Reduces the duration of neutropenia following chemotherapy

 

Ondansetron (Zofran): Prevents nausea

 

Amifostine: Protects against cisplatin toxicity (mucositis, esophagitis) Correct Answer: 2-mercaptoethanesulfonate (MESNA)

 

Which of the following chemotherapeutic agents is the most important in the treatment of patients with osteosarcoma:

 

1) Cyclophosphamide

3) Adriamycin

2) Cisplatin

5) Methotrexate

4) Vincristine

 

Virtually all chemotherapeutic regimens for the treatment of osteosarcoma utilize multidrug combinations. The dose intensity of methotrexate has been found to be the most important factor in disease-free survival.

The most common agents are:

 

 

 

 

 

 

 

 

 

Cyclophosphamide Ifosfamide Cisplatin Adriamycin Actinomycin D Bleomycin Vincristine Etoposide Methotrexate

 

Correct Answer: Methotrexate

 

 

3528. (88) Q9-132:

Which of the following describes the histologic appearance of osteoblastoma:

 

1) Osteoblastomas have a high mitotic rate with atypical mitoses

3) There is a lack of rimming osteoblasts on the bone trabeculae

2) Osteoblastomas show an interlacing network of trabeculae in a loose fibrovascular stroma with prominent vasculature

5) Lace-like osteoid is prominent

4) Multi-nucleated giant cells are a rare finding

 

Osteoblastomas have a characteristic histologic appearance that is best described as an interlacing network of bone trabeculae in a loose fibrovascular stroma with prominent vasculature.

 

Other important features to remember are: Presence of rimming osteoblasts

 

Multi-nucleated giant cells are common.

To help differentiate osteoblastoma from malignant lesions, one should remember that:

 

 

Lace-like osteoid production, which is common in osteosarcomas, is generally not found in osteoblastomas. The nuclei lack pleomorphism.

 

Atypical mitoses are not present.

Correct Answer: Osteoblastomas show an interlacing network of trabeculae in a loose fibrovascular stroma with prominent vasculature

 

Which of the following tumors has been associated with a chromosomal translocation from chromosome 9 to 22:

 

1) Malignant fibrous histiocytoma

3) Parosteal osteosarcoma

2) Fibrosarcoma

5) Periosteal osteosarcoma

4) Extra-skeletal myxoid chondrosarcoma

 

Recently, 2 cartilage tumors have been associated with genetic abnormalities:

Extra-skeletal myxoid chondrosarcoma:Translocation from chromosome 9 to 22 Multiple exostoses:Exostosis genes:

EXT 1 chromosome 8 (8q24)

EXT 2 chromosome 11 (11p13)

EXT 3 chromosome 19 (19q)

Correct Answer: Extra-skeletal myxoid chondrosarcoma

 

 

3530. (90) Q9-134:

Microscopically, aggressive osteoblastomas are similar to conventional osteoblastomas. Which of the following histologic findings distinguishes aggressive osteoblastomas from conventional osteoblastomas:

 

1) Prominent vasculature

3) Epithelioid osteoblasts

2) Loose fibrovascular stroma

5) Peripheral shell of reactive bone bordering soft tissue extension

4) Irregular network of interlacing bone trabeculae

 

'Aggressive osteoblastoma' is a term used to describe osteoblastomas that tend to behave in a more locally aggressive fashion than conventional osteoblastomas. One must remember that 'aggressive osteoblastomas' are still benign lesions and do not metastasize.

 

Histologically, one distinguishes 'conventional osteoblastoma' from 'aggressive osteoblastoma' by the presence of epithelioid osteoblasts. These are plump, round to oval osteoblasts that have a peri-nuclear halo, which is similar to plasma cells.

 

The other features listed as possible answers are common characteristics found in both forms of osteoblastoma: Prominent vasculature

 

Loose fibrovascular stroma

 

Irregular network of interlacing bone trabeculae

 

 

Peripheral shell of reactive bone bordering soft tissue extension Correct Answer: Epithelioid osteoblasts

 

3531. (91) Q9-135:

Osteosarcoma occurs most commonly in which of the following bones:

 

1) Distal femoral metaphysis and proximal humeral metaphysis

3) Distal femoral metaphysis and distal radial metaphysis

2) Distal femoral metaphysis and proximal tibial metaphysis

5) Proximal tibial metaphysis and proximal humeral metaphysis

4) Distal femoral metaphysis and ilium

 

Osteosarcoma most commonly occurs about the knee. The distal femur is the most common site followed by the proximal tibia. These two growth plates account for the majority of the growth in the lower extremity.

 

In order of frequency, other common sites include: Proximal humerus

 

Ilium

 

 

Proximal femur Distal tibia

Correct Answer: Distal femoral metaphysis and proximal tibial metaphysis

 

3532. (92) Q9-137:

Which of the following cartilage tumors has the worst prognosis in regard to survival:

 

1) Grade II chondrosarcoma of the proximal femur

3) Grade 2 chondrosarcoma of the distal femur

2) Grade I chondrosarcoma of the pelvis

5) Grade I chondrosarcoma arising in a pelvic osteochondroma

4) Dedifferentiated chondrosarcoma of the distal femur

 

In cartilage tumors, the histologic grade of the lesion is the most important determinate of long-term disease-free survival. Grade I chondrosarcomas seldom, if ever, metastasize. In contrast, dedifferentiated chondrosarcomas metastasize in 75% to 90% of cases.

 

A study of 67 pelvic chondrosarcomas showed the following rate of metastases:

 

Grade I

0%

Grade II

20%

Grade III

60%

Dedifferentiated

75%

 

Approximate Rate of Metastases

 

 

Correct Answer: Dedifferentiated chondrosarcoma of the distal femur 3533. (93) Q9-138:

Chondrosarcoma rarely occurs in which of the following locations:

 

1) Pelvis

3) Proximal femur

2) Proximal humerus

5) Carpal bones and phalanges

4) Distal femur

 

Chondrosarcomas are common in the pelvis, proximal humerus, and proximal and distal femur. In contrast, chondrosarcomas rarely occur in the hands and feet.

Interestingly, although chondrosarcomas are rare in the hands and feet, the histologic appearance of enchondromas of the hands and feet can be worrisome. Cellular atypia is often accepted in the hands and feet and still classified as a benign enchondroma. However, the same histology in a pelvic or femoral location would be consistent with a medium or high-grade chondrosarcoma.

 

Correct Answer: Carpal bones and phalanges

 

 

3534. (94) Q9-139:

Which of the following pairs of tumors are related as they share a chromosomal translocation between chromosome 11 and chromosome 22 (t [11;22]):

 

1) Chondrosarcoma and osteosarcoma

3) Adamantinoma and osteofibrous dysplasia

2) Malignant fibrous histiocytoma and fibrosarcoma

5) Osteoid osteoma and osteoblastoma

4) Ewingâs tumor and primitive neuroectodermal tumor

 

Ewingâs tumor and primitive neuroectodermal tumor are related as they share a chromosomal translocation between chromosome 11 and chromosome 22 (t [11;22]). A chimeric protein is produced that appears to act as transcriptional activator. This translocation is found in 95% of these tumors.

 

Correct Answer: Ewingâs tumor and primitive neuroectodermal tumor

 

Which of the following factors is a poor prognostic variable in patients with high-grade intra-medullary osteosarcoma:

 

1) Distal femoral location

3) Elevated alkaline phosphatase level

2) Soft tissue extension

5) Male sex

4) Expression of multidrug resistance gene

 

The expression of the multidrug resistance (MDR) gene has been shown to be an adverse survival factor in several studies. MDR-1 codes for p-glycoprotein, a calcium-dependent adenosine triphosphatase, located on the cell membrane. This protein has the ability to pump chemotherapeutic agents out of the cell (especially adriamycin). In a recent study, the presence of this glycoprotein increased the risk of a poor survival in patients by over 3 times. P-glycoprotein has been found in up to 25% of primary tumors and 50% of metastatic tumors.

 

Correct Answer: Expression of multidrug resistance gene

 

 

3536. (96) Q9-141:

Metastases are common in patients with high-grade intramedullary osteosarcoma. The most common sites of metastases are:

 

1) Lungs and lymph nodes

3) Lungs and bone

2) Lungs and liver

5) Lungs and kidney

4) Lungs and brain

 

Patients with high-grade intramedullary osteosarcoma have a high rate of metastases to the lungs (90%) and bone (20%). Approximately 10% to 20% of patients present with metastatic disease in the lungs or bone (Musculoskeletal Tumor Society Stage III).

 

Patients who have negative staging studies (normal technetium bone scan and CT scan of the chest) still have an incidence of 80% to 90% of occult metastases. In the pre-chemotherapy era, the survival in patients with osteosarcoma was only 10% to 20% because there was no method of killing the occult metastases. Now with multi-agent chemotherapy that is delivered pre- and postoperatively, we are able to save over 60% of these patients.

 

Correct Answer: Lungs and bone

 

 

3537. (97) Q9-142:

The risk of developing a sarcoma in an enchondroma or an osteochondroma is approximately:

 

1) Less than 1%

3) 5% to 10%

2) 3% to 5%

5) 15% to 20%

4) 10% to 15%

 

Enchondromas and osteochondromas are the most common benign tumors of the skeleton. The risk of developing a sarcoma within an enchondroma or an osteochondroma is extremely low. The incidence is far below 1%.

Correct Answer: Less than 1%

 

Giant cell tumor of bone is very prone to local recurrence following curettage. What is the approximate local recurrence rate following curettage and methyl methacrylate augmentation:

 

1) 1%

3) 8% to 25 %

2) Less than 5 %

5) 50% to 75 %

4) 25% to 50%

 

Giant cell tumor of bone is very prone to local recurrence. Historically, series in which the tumor was removed by curettage and then the defect reconstructed with methyl methacrylate has local failure rates as high as 40% to 50%. Because of the high recurrence rate, some surgeons advocated resection and reconstruction with allografts or prosthetic devices, especially for recurrent tumors.

 

A new treatment method has been developed that is an extended form of curettage. The principles of treatment are as follows: Wide exteriorization: Removal of the cortex over the lesion

 

 

Curettage with hand curettes Curettage with a power burr

 

 

Adjuvant cauterization with one of the following: Phenol, hydrogen peroxide, or liquid nitrogen Reconstruction of the subchondral surface with autogenous bone graft

 

Methyl methacrylate reconstruction of the defect

 

Internal fixation as necessary: Either Steinmann pins inside the medullary cavity or plate fixation The local recurrence rate in 2 recent series with this technique was 8% (3 of 38) and 25% (15 of 60).

Correct Answer: 8% to 25 %

 

 

3539. (99) Q9-144:

Which of the following radiographic features helps to distinguish aneurysmal bone cyst from unicameral bone cyst:

 

1) Purely lytic appearance

3) Frequently borders the growth plate

2) Frequently metaphyseal

5) Expansion of the bone

4) Eccentric location within the bone

 

 

Unicameral bone cysts have a typical radiographic appearance: Metaphyseal

 

Purely lytic

 

 

Expand the bone in a symmetric fashion Often border the growth plate

 

May have trabeculations in them once they have fractured

One can usually distinguish aneurysmal bone cyst (ABC) from unicameral bone cyst by recognizing the eccentric nature of the ABC. Unicameral bone cysts will generally not expand the bone beyond the width of the physis. In contrast, ABC may markedly expand the cortex. As ABCs expand into the soft tissues, there is generally a thin rim of periosteal bone that outlines the expansion. With aneurysmal bone cyst, the lesion is eccentric. Normal bone is present bordering the lesion. In contrast, in unicameral bone cyst the entire medullary cavity is symmetrically involved.

 

Correct Answer: Eccentric location within the bone

 

 

 

1) Proximal femur and distal

3) Proximal humerus and proximal femur

2) Proximal humerus and distal femur

5) Proximal femur and distal humerus

4) Proximal humerus and distal tibia

 

Unicameral bone cysts most commonly occur in the proximal humerus. The proximal femur is the second most common site. Unicameral bone cysts have a typical radiographic appearance:

 

 

Metaphyseal Purely lytic

 

 

Expand the bone in a symmetric fashion Often border the growth plate

 

 

May have trabeculations in them once they have fractured Correct Answer: Proximal humerus and proximal femur

 

3541. (101) Q9-146:

Which of the following sarcomas characteristically has a translocation between chromosomes 11 and 22:

 

1) Dedifferentiated chondrosarcoma

3) Fibrosarcoma

2) Malignant fibrous histiocytoma

5) Post-irradiation sarcoma

4) Ewingâs tumor

 

Ewingâs tumor has a characteristic translocation between chromosomes 11 and 22 in 95% of cases. The translocation produces a fusion protein between the EWS and Fli-1 genes.

Correct Answer: Ewingâs tumor

 

 

3542. (102) Q9-147:

In which of the following bone tumors is CT scanning superior to MRI:

 

1) High-grade intramedullary osteosarcoma

3) Malignant fibrous histiocytoma

2) Ewingâs tumor

5) Giant cell tumor of bone

4) Osteoid osteoma

 

MRI has replaced CT for the imaging of almost all bone tumors. One exception is osteoid osteoma. Osteoid osteomas are small, lytic lesions (5 mm to 15 mm) that have a dense sclerotic rim. The nidus and its sclerotic rim are best identified with CT scans. The lesions can be detected with the MRI, but they can not be seen as clearly and may be easily missed.

 

High-grade intramedullary osteosarcoma, Ewingâs tumor, malignant fibrous histiocytoma, and giant cell tumor are best imaged with MRI scans.

Correct Answer: Osteoid osteoma

 

 

 

1) Expansile, destructive lesion in the posterior elements

3) Diffuse osteopenia in the vertebral body

2) A 10-mm lytic area with surrounding sclerosis in the posterior elements

5) Destruction of a pedicle on one side

4) Completely collapsed vertebral body with increased width

 

One of the earliest findings in metastatic bone disease is destruction of a pedicle on one side of the vertebral body. Since the pedicle is composed of mostly cortical bone, one can detect the destruction easily on anteroposterior radiograph. In contrast, the lesion can progress through the vertebral body's cancellous bone without prominent bone destruction making detection difficult.

 

The other possible answers describe different entities:

 

Osteoblastoma or aneurysmal bone cyst:

Expansile, destructive lesion in the posterior elements

 

Osteoid osteoma: A 10-mm lytic area with surrounding sclerosis in the posterior

elements

 

Multiple myeloma: Diffuse osteopenia in the vertebral body

 

Eosinophilic granuloma: Completely collapsed vertebral body with increased width Correct Answer: Destruction of a pedicle on one side

 

 

3544. (104) Q9-149:

Which of the following describes the typical radiographic appearance of eosinophilic granuloma in the spine:

 

1) Expansive, destructive lesion in the posterior elements

3) Diffuse osteopenia in the vertebral body

2) A 10-mm lytic area with surrounding sclerosis in the posterior elements

5) Destruction of a pedicle on one side

4) Completely collapsed vertebral body with increased width

 

Eosinophilic granuloma has a characteristic appearance in the spine. There is complete collapse of the vertebra and the vertebral body is often widened. This almost pathognomonic appearance is called 'vertebra plana.' Occasionally, Ewing's tumor or leukemia can have a similar appearance.

 

The other possible answers describe different entities:

 

Osteoblastoma or aneurysmal bone cyst:

Expansile, destructive lesion in the posterior elements

 

Osteoid osteoma: A 10-mm lytic area with surrounding sclerosis in the posterior

elements

 

Multiple myeloma: Diffuse osteopenia in the vertebral body

 

Metastatic bone disease: Destruction of a pedicle on one side Correct Answer: Completely collapsed vertebral body with increased width

 

destruction of about one-third of the cortical bone. The lesion is biopsied and is an eosinophilic granuloma (Langerhansâ cell granulomatosis). The best treatment method would be:

 

1) Wide resection and allograft reconstruction

3) High-dose external beam irradiation and chemotherapy

2) Preoperative chemotherapy followed by wide resection (allograft or prosthesis)

5) Intralesional corticosteroid injection

4) Multi-agent chemotherapy alone

 

Eosinophilic granuloma, or Langerhans cell granulomatosis, is a reactive condition that can often cause marked bone destruction. This condition usually occurs in children and young to middle aged adults.

The characteristic radiographic finding is a well-demarcated punched out lesion. The femur, humerus, and flat bones are common sites. Occasionally, patients may present with a very aggressive lesion with a permeative pattern of bone destruction, cortical bone destruction, and a soft tissue mass.

 

Eosinophilic granuloma is a self-limited disorder. There have been many different treatments proposed such as biopsy only, curettage and bone grafting, and low-dose irradiation. Most recently, intralesional cortisone injection has become the standard treatment for most lesions.

 

Correct Answer: Intralesional corticosteroid injection

 

 

3546. (106) Q9-151:

Which of the following is the most prominent feature of the presentation of children with osteosarcoma:

 

1) Painless mass

3) Pain with activity, relieved by rest

2) Recurrent atraumatic effusions

5) Pain following sports, especially running and jumping

4) Pain that may begin as intermittent and progresses to constant

 

Pain is the most prominent feature of high-grade malignant tumors in children and adults. The pain is a deep-seated pain and is usually well localized when the lesion is in the extremity. The pain begins as intermittent pain and then progresses to constant pain. The pain often occurs at night and at rest. As the tumor grows, the pain becomes constant and relentless.

 

Pain from malignant bone tumors: Is dull or sharp

 

Is deep-seated

 

Occurs at rest and at night

 

Begins as intermittent discomfort and progresses to constant

Correct Answer: Pain that may begin as intermittent and progresses to constant

 

 

 

1) Proximal femur

3) Proximal humerus

2) Distal femur

5) Phalanges of the hand

4) Scapula

 

Enchondromas are hyaline cartilage tumors that occur in the medullary cavity. They are usually asymptomatic. They do not expand or cause major erosions of the cortical bone in long bones; however, in small bones enchondromas may cause cortical expansion.

 

In order of occurrence, the most frequent sites of enchondromas include: Small bones of the hands and feet (hand to feet ratio 7:1)

 

Femur

 

Proximal humerus

 

Less common: forearm, tibia, and fibula

 

It is rare for enchondromas to occur in the following sites: Pelvis

 

 

 

Ribs Scapula Vertebra

 

Correct Answer: Scapula

 

 

 

1) Scattered stippling

3) Cloudy opacities that vary in size, shape, and density

2) Ring-like

5) Stippling with multiple rings and arcs

4) Flocculent

 

It is important to be able to recognize the mineralization pattern of lesions in both bone and soft tissue. Mineralization may occur in osteoid and hyaline cartilage. One can broadly say that the mineralization is bone formation, cartilage calcification, and amorphous calcification.

 

 

The following table is useful to remember when analyzing radiographs:

 

 

 

Mineralization pattern

 

Lesions

 

 

 

 

 

Hyaline cartilage calcification

 

Fine stipples*

Rings, arcs, commas

 

Enchondroma

Low-grade intramedullary chondrosarcoma

Dedifferentiated chondrosarcoma Osteochondroma

Secondary surface chondrosarcoma Synovial chondromatosis

 

 

 

 

 

Bone formation

 

Cloud- or ivory-like (may be trabecular pattern in the bone)

 

Bone islands Heterotopic ossification Osteoblastoma

Osteoid osteoma

High-grade intramedullary osteosarcoma

Low grade intramedullary osteosarcoma

 

 

 

 

 

Amorphous calcification

 

Nodular and flocculent† (multiple areas in most lesions)

 

Tumorous calcification Calcific tendinitis

 

* If one were to paint fine stippling, one would use a brush with a fine tip.

 

† If one were to paint the amorphous calcifications, one would use the main body of the brush, not a fine tip. Correct Answer: Cloudy opacities that vary in size, shape, and density

and external beam irradiation. He now has pain in the proximal humerus and his radiographs show a lytic lesion with a small area of cortical bone destruction. The most likely diagnosis:

 

1) Stress fracture

3) Osteomyelitis

2) Radiation osteitis

5) Post-irradiation sarcoma

4) Recurrent Ewingâs tumors

 

Patients who have been treated with irradiation may develop two principal problems in their skeleton â insufficiency, or stress fractures, and post-irradiation sarcomas.

Fortunately, post-irradiation sarcomas are rare, but they may occur in a patient who has been cured of their cancer. Patients may have received either radiation alone or a combination of chemotherapy and radiation. The latency period (interval between the completion of the radiation therapy and onset of the sarcoma) varies between 5 to 50+ years. There have been some reports with post-irradiation sarcomas occurring within 2 to 5 years.

 

The symptoms of a post-irradiation sarcoma are pain and swelling. Occasionally, patients present with a mass only.

The key radiographic feature of a post-irradiation sarcoma is cortical bone destruction and the presence of a soft tissue mass. The cortical bone destruction may be appreciated on plain radiographs alone or with CT or MRI scans.

NOTE: When patients with a history of cancer and irradiation present with skeletal pain and normal radiographs, the clinician should search further with a cross-sectional imaging modality, such as CT or MRI scans.

Correct Answer: Post-irradiation sarcoma

 

 

3550. (111) Q9-156:

Enchondromas may commonly occur in all of the following sites except:

 

1) Proximal femur

3) Proximal humerus

2) Distal femur

5) Phalanges of the hand

4) Vertebra

 

Enchondromas are hyaline cartilage tumors that occur in the medullary cavity. They are usually asymptomatic. They do not expand long bones or cause major erosions of the cortical bone. In contrast, enchondromas may cause cortical expansion in small bones.

 

In order of occurrence, the most frequent sites of enchondromas include: Small bones of the hands and feet (hand to feet ratio 7:1)

 

Femur

 

Proximal humerus

 

Less common: forearm, tibia, and fibula

 

It is rare for enchondromas to occur in the following sites: Pelvis

 

 

 

Ribs Scapula Vertebra

 

Correct Answer: Vertebra

 

Which of the following bone tumors are sensitive to radiation (an appropriate dose will kill the tumor with acceptable side effects to normal tissues):

 

1) Lymphoma, myeloma, and metastatic carcinomas

3) Dedifferentiated chondrosarcoma, malignant fibrous histiocytoma, and Ewingâs tumor

2) Metastatic bone disease, myeloma, and malignant fibrous histiocytoma

5) Clear cell chondrosarcoma, mesenchymal chondrosarcoma, and Ewingâs tumor

4) Parosteal osteosarcoma, periosteal osteosarcoma, and high-grade surface osteosarcoma

 

The lesions that are sensitive to irradiation include:

 

 

Metastatic carcinomas (renal cell carcinoma is more prone to recur following irradiation) Multiple myeloma

 

Lymphoma

Most sarcomas are relatively radioresistant.

Correct Answer: Lymphoma, myeloma, and metastatic carcinomas

 

 

 

3552. (113) Q9-158:

Which of the following benign soft tissue tumors is very prone to local recurrence after surgical removal:

 

1) Nodular fasciitis

3) Angiolipoma

2) Lipoma

5) Extra-abdominal desmoid tumor

4) Schwannoma

 

Extra-abdominal desmoid tumors are very prone to local recurrence. Despite adequate surgery, these lesions may recur in up to 50% of patients. This lesion has an infiltrative growth pattern with prominent invasion of surrounding muscle. Local recurrences may be treated with repeat excision, external beam irradiation, or low-dose chemotherapy. If the lesion does not display active and progressive growth, then the local recurrence can be observed.

 

In contrast, many benign lesions can be treated with simple excision with a very low risk of local recurrence. Examples of this self-limited growth pattern include:

 

 

 

 

Nodular fasciitis Lipoma Angiolipoma Schwannoma

 

Correct Answer: Extra-abdominal desmoid tumor

 

 

3553. (115) Q9-160:

In patients with localized high-grade intramedullary osteosarcoma (Stage II), what is the most important factor in regard to long-term disease-free survival:

 

1) Sex of the patient

3) Size of the initial tumor

2) Age of the patient

5) Presence or absence of soft tissue extension

4) Response to preoperative chemotherapy

 

The most important determinant of long-term disease-free survival is response to preoperative chemotherapy. Multiple histologic sections are analyzed to determine the amount of tumor necrosis. The most favorable survival occurs in those patients with 99% or 100% tumor kill. When less than 90% tumor necrosis is achieved, the response is judged to be poor and there is a significant risk for both local recurrence and systemic failure.

 

 

Sex, size, age, and presence of soft tissue extension are not strong predictors of survival. Correct Answer: Response to preoperative chemotherapy

In which of the following malignant tumors is bone marrow aspiration part of the initial staging evaluation:

 

1) High-grade intramedullary osteosarcoma

3) Clear cell chondrosarcoma

2) Dedifferentiated chondrosarcoma

5) Malignant fibrous histiocytoma

4) Ewingâs tumor

 

The staging evaluation is predicated on the most common site of metastases of various bone tumors. The most common site for virtually all high-grade bone tumors is to the lungs. While most high-grade sarcomas can also metastasize to bone, the bone metastases occur later rather than earlier.

 

In contrast, in Ewing's tumor the bone metastases occur early and are commonly present at initial presentation. Because of the propensity for early bone metastases, bone marrow aspiration is part of the staging evaluation for patients with Ewingâs tumor. These bone metastases may be present even when the bone scan is normal.

 

Correct Answer: Ewingâs tumor

 

 

3555. (117) Q9-162:

A 64-year-old man has a 6-month history of back pain. Serum laboratory studies show a hemoglobin level of 11 mg/dL and an erythrocyte sedimentation rate of 110 mm/hr. Based on these studies, the most likely diagnosis is:

 

1) Metastatic breast cancer

3) Multiple myeloma

2) Metastatic lung cancer

5) Fibrous dysplasia

4) Pagetâs disease

 

Patients with multiple myeloma often present with both anemia and an elevated sedimentation rate. The low hemoglobin level and elevated erythrocyte sedimentation rate occur in up to two-thirds of patients. Serum protein and urine protein electrophoresis are very sensitive tests to detect the myeloma protein and should be ordered whenever the clinician suspects multiple myeloma.

 

Correct Answer: Multiple myeloma

 

In which of the following bone tumors is curettage performed as the primary method of treatment:

 

1) Dedifferentiated chondrosarcoma

3) Adamantinoma

2) Malignant fibrous histiocytoma

5) Well-differentiated intramedullary osteosarcoma

4) Giant cell tumor

 

There are four different surgical margins that can be achieved in oncologic surgery:

Intralesional: The plane of surgery goes through the tumor.

Marginal: The plane of surgery goes through the reactive zone of the lesion (the reactive zone contains edema, tumor cells, fibrous tissue, and inflammatory cells).

Wide: The plane of dissection goes through normal tissue.

Radical: The entire anatomic compartment of the lesion is removed.

Curettage is reserved for benign tumors or metastatic disease, in which radiation will be used to control whatever tumor cells have been left benign. Giant cell tumors of bone are treated with curettage. The curettage is extensive with exteriorization (removing the cortex off the tumor), followed by curetting the tumor with hand instruments and power tools. There is often a large defect left following the curettage.This defect is usually reconstructed with methyl methacrylate.

 

The following lesions are commonly treated with curettage:

Giant cell tumor Chondroblastoma Aneurysmal bone cyst Osteoid osteoma

 

 

In contrast, malignant bone tumors are usually removed with a wide or radical surgical margin. Correct Answer: Giant cell tumor

 

3557. (119) Q9-164:

Which of the following features of soft tissue sarcomas portends a favorable prognosis:

 

1) Deep location (below the fascia)

3) High-grade histology

2) Superficial location (above the fascia)

5) Lymph node involvement

4) Large size (>10 cm)

 

 

The following features of soft tissue sarcomas are associated with a more favorable prognosis: Small size (<5 cm)

 

Low-grade

 

Location above the fascia Poor prognostic factors include:

 

 

Large size (>5 cm) High-grade

 

 

Location below the fascia Presence of regional metastases

 

Patients who have large, deep high-grade sarcomas have a high risk (up to 40% to 60%) of pulmonary metastases. Correct Answer: Superficial location (above the fascia)

In which of the following conditions is polyostotic involvement common:

 

1) Giant cell tumor

3) Chondromyxoid fibroma

2) Chondroblastoma

5) Desmoplastic fibroma

4) Fibrous dysplasia

 

Polyostotic involvement is common in a number of bone tumors. A useful differential diagnosis of multiple lesions is as follows in both young and old patients:

 

 Benign Malignant

 

Younger Patients (age 10 to 40 years)

 

Older Patients (age 40 to 80+ years)

 

Eosinophilic granuloma (Langerhans cell granulomatosis)

Fibrous dysplasia Multiple exostoses

Multiple enchondromatosis

 

Paget's disease Fibrous dysplasia Bone infarcts Hyperparathyroidism Mastocytosis

 

Epithelioid hemangioendothelioma Metastases:

Neuroblastoma/Wilm's tumor (young children

< 10 years old) Melanoma

 

Metastatic carcinomas Multiple myeloma Lymphoma

Epithelioid hemangioendothelioma

Correct Answer: Fibrous dysplasia

 

 

3559. (121) Q9-166:

Which of the following radiographic features helps in differentiating a chondrosarcoma from an enchondroma:

 

1) Stippled and ring-like calcification

3) Expansion of the cortex in a small bone

2) Small endosteal erosions (< 50% cortical width)

5) Absence of matrix calcification

4) Cortical perforation and cortical thickening

 

Enchondromas and chondrosarcomas are both intramedullary cartilage lesions. In the adult patient, enchondromas do not grow; they are inactive lesions. When an intramedullary cartilage lesion grows and destroys the bone, the lesion is a chondrosarcoma.

 

The hallmark feature of almost all chondrosarcomas is involvement of the cortical bone. The cortical bone involvement can manifest in several different ways:

 

Erosion of the cortex:

 

Endosteal erosion occurs slowly as the tumor grows; erosions that are greater than 50% of the thickness of the cortex are suggestive of a chondrosarcoma

 

 

 

Thickening of the cortex:

 

Most chondrosarcomas are low-grade lesions. The tumor grows slow and may gradually distort the cortex. With low-grade lesions, the endosteal surface may be resorbed with concomitant periosteal new bone formation of the periosteal surface. The cortex is then thickened by the endosteal erosion of the chondrosarcoma.

 

 

 

Destruction of the cortex:

 

Some chondrosarcomas simply destroy the cortex as they grow and the lesion then extends into the soft tissues. When there is complete bone destruction, these chondrosarcomas are usually of a higher grade.

Correct Answer: Cortical perforation and cortical thickening

 

Which of the following osteosarcomas has minimal osteoid production when examined histologically:

 

1) Parosteal osteosarcoma

3) Well-differentiated intramedullary osteosarcoma

2) Periosteal osteosarcoma

5) Telangiectatic osteosarcoma

4) High-grade intramedullary osteosarcoma

 

Telangiectatic osteosarcomas are the one form of osteosarcoma where there may be minimal or no osteoid production by the tumor cells. The radiographs typically show a destructive, purely lytic lesion that is often expansile.

On low power viewing, there are usually large blood-filled spaces. The low-power appearance may be very similar to that seen with aneurysmal bone cyst. On high-power viewing, the cells that are in the cyst lining are malignant. The nuclei show pleomorphism and atypical features. There are often bizarre mitotic figures.

 

Correct Answer: Telangiectatic osteosarcoma

 

 

3561. (123) Q9-168:

In which of the following tumors are viral-like inclusion bodies found in the osteoclasts when studied with electron microscopy:

 

1) Fibrous dysplasia

3) Giant cell tumor

2) Pagetâs disease

5) Chondromyxoid fibroma

4) Chondroblastoma

 

In Pagetâs disease, there are viral-like inclusion bodies found in the osteoclasts. Because of these particles, some investigators believe Pagetâs disease may be caused by a slow virus. The findings that support a slow virus include: the long disease course, single organ system, and absence of fever. The viral-like particles may belong to the paramyxovirus family.

 

Correct Answer: Pagetâs disease

 

 

3562. (124) Q9-169:

Which of the following groups of carcinomas commonly produce sclerotic metastases to bone:

 

1) Breast and prostate

3) Lung and thyroid

2) Lung and kidney

5) Kidney and bladder

4) Lung and bladder

 

Bone metastases are usually of three patterns: predominantly lytic, mixed lytic-blastic, and purely blastic. Common Patterns

Purely lytic Lung

Kidney Thyroid

Mixed lytic-blastic Breast

Purely blastic Prostate

Breast

Correct Answer: Breast and prostate

 

Which of the following lesions is typically a Stage 1 lesion according to the staging system of the Musculoskeletal Tumor Society:

 

1) Giant cell tumor

3) Non-ossifying fibroma

2) Chondroblastoma

5) Osteoid osteoma

4) Osteoblastoma

 

Stage 1 lesions are inactive, or latent, lesions. These lesions do not cause symptoms and generally do not grow. There is no risk of pathologic fracture or other problems.

Non-ossifying fibroma, or fibrous cortical defect, is a developmental process that is usually discovered as an incidental finding. The lesions are usually small and affect less than 50% of the diameter of the cortical bone. When these lesions are small (< 50% of the diameter of the bone) and asymptomatic, patients are not restricted in their activity. A new radiograph is obtained at 6 to 12 months to ensure that the lesion is not enlarging.

 

Some patients with non-ossifying fibromas have active lesions that grow and cause symptoms. Patients occasionally sustain fractures following minor trauma. In the scenarios that involve pain or fracture, the non-ossifying fibroma would be classified as a Stage 2 lesion.

 

Correct Answer: Non-ossifying fibroma

 

 

3564. (126) Q9-171:

Which of the following bone lesions is typically treated by intralesional curettage:

 

1) Dedifferentiated chondrosarcoma

3) Adamantinoma

2) Malignant fibrous histiocytoma

5) Well-differentiated intramedullary osteosarcoma

4) Aneurysmal bone cyst

 

There are four different surgical margins that can be achieved in oncologic surgery:

Intralesional: The plane of surgery goes through the tumor.

Marginal: The plane of surgery goes through the reactive zone of the lesion (the reactive zone contains edema, tumor cells, fibrous tissue, and inflammatory cells).

Wide: The plane of dissection goes through normal tissue.

Radical: The entire anatomic compartment of the lesion is removed.

Curettage is reserved for benign tumors or malignant tumors, in which radiation will be used to control whatever tumor cells have been left benign. Giant cell tumors of bone are treated with curettage. The curettage is extensive with exteriorization (removing the cortex off the tumor), followed by curetting the tumor with hand instruments and power tools. There is often a large defect left following the curettage.This defect is usually reconstructed with methyl methacrylate.

 

The following lesions are commonly treated with curettage:

Giant cell tumor Chondroblastoma Aneurysmal bone cyst Osteoid osteoma

 

 

In contrast, malignant bone tumors are usually removed with a wide or radical surgical margin. Correct Answer: Aneurysmal bone cyst

Which of the following bone tumors is typically treated with intralesional curettage:

 

1) Dedifferentiated chondrosarcoma

3) Adamantinoma

2) Malignant fibrous histiocytoma

5) Well-differentiated intramedullary osteosarcoma

4) Giant cell tumor

 

There are four different surgical margins that can be achieved in oncologic surgery:

Intralesional: The plane of surgery goes through the tumor.

Marginal: The plane of surgery goes through the reactive zone of the lesion (the reactive zone contains edema, tumor cells, fibrous tissue, and inflammatory cells).

Wide: The plane of dissection goes through normal tissue.

Radical: The entire anatomic compartment of the lesion is removed.

Curettage is reserved for benign tumors or malignant tumors in which radiation will be used to control whatever tumor cells have been left benign. Giant cell tumors of bone are treated with curettage. The curettage is extensive with exteriorization (removing the cortex off the tumor), followed by curetting the tumor with hand instruments and power tools. There is often a large defect left following the curettage. This defect is usually reconstructed with methyl methacrylate.

 

The following lesions are commonly treated with curettage:

Giant cell tumor Chondroblastoma Aneurysmal bone cyst Osteoid osteoma

 

Correct Answer: Giant cell tumor

 

 

3566. (128) Q9-173:

Which of the following carcinomas typically produces purely lytic bone metastases:

 

1) Lung and breast

3) Bladder and thyroid

2) Lung and prostate

5) Breast and prostate

4) Kidney and lung

 

Bone metastases are usually of three patterns: predominantly lytic, mixed lytic-blastic, and purely blastic. Common Patterns

Purely lytic Lung

Kidney Thyroid

Mixed lytic-blastic Breast

Purely blastic Prostate

Breast

Correct Answer: Kidney and lung

 

In which of the following metastatic carcinomas is preoperative embolization useful to decrease bleeding in large lesions with soft tissue extension:

 

1) Adenocarcinoma of the lung

3) Renal cell carcinoma

2) Breast cancer

5) Small cell cancer of the lung

4) Prostate cancer

 

Renal cell carcinomas can be very vascular lesions. There may be thousands of small and large blood vessels supplying the lesion. Lesion with marked expansion of the bone or large soft tissue masses are especially at risk for massive bleeding.

Preoperative embolization is an excellent method of reducing intraoperative bleeding. Patients are usually embolized once within 1 week of surgery. Occasionally, a very vascular lesion may require more than one separate angiographic procedures to complete the embolization.

 

Correct Answer: Renal cell carcinoma

 

 

3568. (130) Q9-175:

Which of the following lesions is high signal on T1 weighted images and moderate signal on T2 weighted images:

 

1) Malignant fibrous histiocytoma

3) Synovial sarcoma

2) Extra-abdominal desmoid tumor

5) Ganglion

4) Lipoma

 

Lipomas are composed of mature adipose tissue. Mature adipose tissue is high signal on T1 weighted images and moderate on T2 weighted images.

 

It is important to remember the appearances of common tissues on both T1 and T2 weighted images:

 

Â

 

T1 weighted

 

T2 weighted

 

 

 

 

 

Fat

 

High

 

Moderate

 

 

 

 

 

Tendons

 

Low

 

Low

 

 

 

 

 

Ligaments

 

Low

 

Low

 

 

 

 

 

Fascial layers

 

Low

 

Low

 

 

 

 

 

Cortical bone

 

Low

 

Low

 

 

 

 

 

Muscle

 

Moderate

 

Moderate

 

 

 

 

 

Normal marrow

 

High

 

Moderate

 

 

 

 

 

Soft tissue sarcomas

 

Low

 

High

 

 

 

 

 

Fluid (ganglions, effusions)

 

Low

 

High

 

 

Pigmented villonodular synovitis will often have signal drop out (dark black) on gradient echo sequences. Correct Answer: Lipoma

Which of the following describes the histologic features of Ewingâs tumor:

 

1) A mixed population of inflammatory cells

3) A population of bi-lobed cells with red granules and Langerhansâ cells

2) A single population of small blue cells

5) Numerous cells with eccentric nuclei, clock-face chromatin, and a prominent peri-nuclear halo

4) Spindle cells with pleomorphism producing osteoid

 

Ewing's tumor is an anaplastic tumor that is composed of a single population of small blue cells on hematoxylin and eosin staining. The cells do not produce a matrix such as bone, cartilage, or fibrous tissue. Because there is no matrix, the tissue is very loose (may not stay together when removed) and may resemble infection (pus-like substance).

 

The other possible answers describe common bone lesions:

 

Infection: A mixed population of inflammatory cells

 

Eosinophilic granuloma:

 

A population of bi-lobed cells with red granules (eosinophils) and Langerhans cells

 

Osteosarcoma: Spindle cells with pleomorphism producing osteoid

 

Myeloma: Numerous cells with eccentric nuclei, clock-face chromatin, and a prominent peri-nuclear halo

Correct Answer: A single population of small blue cells

 

 

3570. (134) Q9-179:

Pagetâs disease is most common in which of the following countries:

 

1) France

3) Italy

2) United Kingdom

5) Russia

4) Spain

 

Pagetâs disease is very common in Europe. The highest incidence is in the United Kingdom. It is less common in Scandinavia, France, Italy, Central Europe, and Russia (Dorfman). Pagetâs disease is also common in Australia and New Zealand, secondary to those countriesâ ancestries coming from the United Kingdom. As patients age, Pagetâs disease can be found in up to 3% to 5 % of the population.

 

Correct Answer: United Kingdom

 

Which of the following radiographic descriptions describes the features of sarcoma occurring in Pagetâs disease:

 

1) Irregular increased density with a cotton wool-like appearance with interspersed radiolucency

3) Large areas of bone sclerosis

2) Bone is enlarged with prominent deformity

5) Osteolytic area with cortical bone destruction

4) Patchy sclerosis

 

Paget's disease has very characteristic radiographic patterns. The disease is divided into three phases: lytic, mixed lytic-sclerotic, and sclerotic phase.

In the sclerotic and mixed phase, the radiographic findings are easy to recognize:

Sclerotic phase

Irregular increased density with cotton wool-like appearance with interspersed radiolucency Bones are usually enlarged

There may be prominent deformity

In long bones, the disease begins at one end and proceeds to the other

Intermediate phase Patchy sclerosis

Lytic phase

In the lytic phase, there may be purely radiolucent findings There is often mild expansion of the bone

The cortices are thinned but intact

When a sarcoma has developed within the bone, there is usually cortical bone destruction and a major lytic focus. Periosteal reaction is usually absent and there is often a large soft tissue mass.

Correct Answer: Osteolytic area with cortical bone destruction

 

 

3572. (136) Q9-181:

Enchondromas may commonly occur in all of the following sites except:

 

1) Proximal femur

3) Proximal humerus

2) Distal femur

5) Phalanges of the hand

4) Pelvis

 

Enchondromas are hyaline cartilage tumors that occur in the medullary cavity. They are usually asymptomatic. They do not expand long bones or cause major erosions of the cortical bone. They may, however, cause cortical expansion in small bones.

 

In order of occurrence, the most frequent sites of enchondromas include: Small bones of the hands and feet (hand to feet ratio 7:1)

 

Femur

 

Proximal humerus

 

Less common: forearm, tibia, and fibula

 

It is rare for enchondromas to occur in the following sites: Pelvis

 

 

 

Ribs Scapula Vertebra

Correct Answer: Pelvis

 

All of the following histologic features are typical of enchondromas except:

 

1) Low cellularity

3) Lobular architecture

2) Small, round nuclei

5) Occasional binucleate cells

4) Prominent myxoid change

 

Enchondromas usually do not display active growth either radiographically or histologically. This intramedullary lesion does not involve the cortex in long tubular bones; there is no major cortical erosion, cortical thickening, or cortical breakthrough.

 

Histologically, enchondromas have the following features:

Low cellularity Small, round nuclei

Occasional binucleate cells

Individual chondrocytes are small and lie in lacunae

 

Foci of fine or coarse calcifications or enchondral bone can be seen Prominent myxoid change is not seen.

 

Small focal areas of myxoid change can be seen in enchondromas of the hand.

 

Enchondromas of the hands/feet: Enchondromas in these locations may show hyper-cellularity, atypical nuclear features, and small amounts of myxoid change yet still be benign lesions.

Correct Answer: Prominent myxoid change

 

 

3574. (138) Q9-183:

A 48-year-old woman has an enchondroma in the proximal humerus. She has pain with overhead activity and her shoulder hurts at night when she lies on it. The radiographs show no cortical erosions, cortical thickening, or other worrisome changes.

Management at this point should be:

 

1) Open incisional biopsy

3) Curettage and bone grafting

2) Needle biopsy through the deltoid muscle

5) Observation with new radiographs in 3 to 6 months

4) Wide resection and allograft prosthetic reconstruction

 

Enchondromas are generally treated with observation. They are common and many patients will have symptoms secondary to other entities, such as rotator cuff tendinitis, biceps tendinitis, greater trochanteric bursitis, and other overuse and degenerative conditions. When an intramedullary hyaline cartilage lesion looks innocent on plain radiographs â no major endosteal erosions, cortical breakthrough, or thickening â the lesion is best regarded as an enchondroma. In the asymptomatic patient or the patient who has pain secondary to tendinitis or other condition, observation should be chosen rather than surgical treatment. New radiographs are taken usually at 3 to 6 months and again at 6 months. If the lesion has not changed on 3 series of radiographs over 12 to 24 months, the lesion can be safely regarded as an enchondroma.

 

Correct Answer: Observation with new radiographs in 3 to 6 months

 

The most common site of periosteal chondromas is:

 

1) Proximal femur

3) Proximal humerus

2) Distal femur

5) Proximal tibia

4) Hand metacarpals

 

Periosteal chondromas are rare lesions. Over 50% of the time, they are located in the proximal humerus. Other locations include the distal femur, proximal tibia, and the hand.

 

Periosteal chondromas have typical radiographic appearances: They are small, usually 3 cm to 5 cm.

 

 

 

They sit on the top of the bone in an excavation in the cortex. There is a buttress of periosteal bone on either side of the lesion. About one-third of radiographs show mineralization from within.

 

 

 

 

 

 

 

 

 

 

Correct Answer: Proximal humerus 3576. (141) Q9-192:

 

Figure 1 Figure 2 Figure 3 Figure 4

 

 

 

Figure 5

A 15-year-old boy presents with a 9-month history of severe thigh pain. The pain is most severe at night and aspirin completely relieves the pain. The plain radiograph is shown in Figure 1 and the CT scan in Figure 2. The most likely diagnosis is:

 

1) Stress fracture

3) Parosteal osteosarcoma

2) Osteomyelitis

5) Osteoblastoma

4) Osteoid osteoma

 

The plain radiograph shows a heavily mineralized lesion on the medial aspect of the femoral shaft (Figure 3). The periosteal reaction is mature without evidence of a lytic focus. The heavy mineralization is suggestive of an osteoid osteoma, but the diagnosis cannot be made without identifying the nidus. The CT scan shows a small, 8-mm lytic focus in the medial cortex. Heavy periosteal new bone surrounds the lesion (Figure 4). When comparing the CT scans of the left and right femurs, one can see the abnormal shape of the medial aspect of the femur with a lytic focus located in the medial femoral cortex (Figure 5).

 

 

The essential features of osteoid osteoma are: Small lytic nidus is between 5 mm and 10 mm.

 

 

Nidus is surrounded by heavy periosteal new bone formation. Often, CT scans are the only way to demonstrate the nidus.

Correct Answer: Osteoid osteoma

 

 

 

 

 

Figure 1

A 15-year-old boy presents with a 9-month history of severe thigh pain. The pain is most severe at night and aspirin completely relieves the pain. The plain radiograph is shown in Figure 1. The imaging study of choice to establish the diagnosis for this patient would be:

 

1) Magnetic resonance imaging (MRI) scan

3) Single proton emission computerized tomography (SPECT) scan

2) Technetium bone scan

5) High-resolution ultrasound

4) Computerized tomography (CT) scan

 

The history of severe night pain and pain responsive to nonsteroidal anti-inflammatory medications suggests that the patient has an osteoid osteoma. Osteoid osteomas are small tumors, with the nidus being the essential component of the lesion. The nidus is small (5 mm to 10 mm) and is often surrounded by heavy periosteal new bone. The center of the nidus is often mineralized. CT scans are the best method to identify the lytic focus embedded in the cortical bone.

 

A MRI scan is a superb method to identify bone and soft tissue neoplasms. However, CT scanning is the best method to identify osteoid osteomas. MRI is less sensitive and specific than CT because the nidus may be mineralized in the center causing it to blend with and not be easily distinguished from the periosteal bone.

 

Technetium bone scanning is a sensitive technique used to detect increased activity in an osteoid osteoma, but when one recognizes the increased activity, the increase is non-specific. The increased activity may be secondary to a stress fracture, osteomyelitis, benign or malignant neoplasm.

 

Correct Answer: Computerized tomography (CT) scan

 

 

 

 

 

Figure 1 Figure 2 Figure 3 Figure 4

 

 

 

Figure 5

A 15-year-old boy presents with a 9-month history of severe thigh pain. The pain is most severe at night. He is unable to concentrate in school due to the pain. The plain radiograph is shown in Figure 1 and the CT scan in Figure 2. The most appropriate initial treatment method would be:

 

1) Physical therapy

3) Anti-inflammatory medication

2) Needle or open biopsy

5) En bloc resection and reconstruction

4) Observation

 

The radiograph and CT scan show the classic features of an osteoid osteoma. As many as 50% of patients can be successfully treated with the initial treatment of nonsteroidal anti-inflammatory drugs (Kniesel and Simon).

If nonoperative treatment fails, surgical therapy is aimed at removing the nidus completely. This can be done percutaneously with CT guided radiofrequency ablation or surgical removal with burring of the bed of the osteoid osteoma following removal.

 

To answer this question correctly, one must be able to recognize the classic features of osteoid osteoma: Small lytic nidus is between 5 mm and 10 mm.

 

 

Nidus is surrounded by heavy periosteal new bone formation. Often, CT scans are the only way to demonstrate the nidus.

The plain radiograph shows a heavily mineralized lesion on the medial aspect of the femoral shaft (Figure 3). The periosteal reaction is mature without evidence of a lytic focus. The heavy mineralization is suggestive of an osteoid osteoma, but the diagnosis cannot be made without identifying the nidus. The CT scan shows a small, 8-mm lytic focus in the medial cortex. Heavy periosteal new bone surrounds the lesion (Figure 4). When comparing the CT scans of the left and right femurs, one can see the abnormal shape of the medial aspect of the femur with a lytic focus located in the medial femoral cortex (Figure 5).

 

Correct Answer: Anti-inflammatory medication

 

 

 

 

Figure 1 Figure 2 Figure 3 Figure 4

 

 

 

Figure 5 Figure 6

A 10-year-old boy injured his leg playing baseball. He now has pain whenever he bears weight on his extremity. He has no rest pain or other medical problems. The plain radiographs are shown in Figures 1 and 2, and a biopsy specimen is shown in Figure

3. The most likely diagnosis is:

 

1) Giant cell tumor

3) Non-ossifying fibroma

2) Aneurysmal bone cyst

5) Ewingâs tumor

4) Osteosarcoma

 

The plain radiograph shows the distinctive features (Figures 4 and 5) of a non-ossifying fibroma.

Radiographic features of a non-ossifying fibroma:

 

 

Metaphyseal Eccentric

 

Well-defined sclerotic rim

 

Thinned, but intact, overlying cortex

The biopsy specimen shows the typical characteristics (Figure 6):

 

 

Fibroblasts Giant cells

 

 

Foam-filled histiocytes Hemosiderin pigment

Treatment:

When asymptomatic, non-ossifying fibromas are treated with observation. This lesion is painful and at risk for fracture because of the activity-related pain. Treatment is curettage and bone grafting.

Correct Answer: Non-ossifying fibroma

 

 

 

 

Figure 1 Figure 2 Figure 3 Figure 4

A 10-year-old boy injured his leg playing baseball. He now has pain whenever he bears weight on his extremity. He has no rest pain or other medical problems. The plain radiographs are shown in Figures 1 and 2, and a biopsy specimen is shown in Figure

3. The most appropriate treatment method would be:

 

1) Preoperative chemotherapy and wide resection

3) Aspiration and methyl prednisolone acetate injection

2) Below-knee amputation

5) Debridement, methyl methacrylate antibiotic beads, and intravenous antibiotics

4) Curettage and bone grafting

 

To answer this question, one must first make the correct diagnosis and then select a treatment for that diagnosis. The radiographic and histologic features presented are most consistent with non-ossifying fibroma.

Radiographic features of a non-ossifying fibroma:

 

 

Metaphyseal Eccentric

 

Well-defined sclerotic rim

 

Thinned, but intact, overlying cortex

The biopsy specimen shows the typical characteristics (Figure 6):

 

 

Fibroblasts Giant cells

 

 

Foam-filled histiocytes Hemosiderin pigment

Treatment:

When asymptomatic, non-ossifying fibromas are treated with observation. This lesion is painful and at risk for fracture because of the activity related pain. Treatment is curettage and bone grafting.

The other possible answers are for distinct entities:

Preoperative chemotherapy and wide resection â osteosarcoma Below-knee amputation â unresectable sarcoma

Aspiration and methyl prednisolone acetate injection â unicameral bone cyst, eosinophilic granuloma Debridement, methyl methacrylate antibiotic beads, and intravenous antibiotics â osteomyelitis

Correct Answer: Curettage and bone grafting

 

 

Figure 1 Figure 2 Figure 3

A 10-year-old boy injured his leg playing baseball. He now has pain whenever he bears weight on his extremity. He has no rest pain or other medical problems. The plain radiographs are shown in Figures 1 and 2, and a biopsy specimen is shown in Figure

3. What would be the appropriate stage according to the system of the Musculoskeletal Tumor Society:

 

1) Stage 1

3) Stage 3

2) Stage 2

5) Stage III

4) Stage II

 

The radiographic and histologic features are consistent with non-ossifying fibroma. Because the lesion is symptomatic, the lesion is active. Non-ossifying fibromas have a low local recurrence rate and they never threaten the involved joint. Hence, this would be a Stage 2 lesion.

 

The Surgical Staging System of the Musculoskeletal Tumor Society is a useful system to both predict prognosis and plan treatment. The system for benign lesions is divided into three groups: inactive (latent), active, and aggressive:

 

Stage 1: Inactive (latent)

Stage 2: Active

 

Stage 3:

 

Refers to lesions that are not causing pain and show no evidence of active growth. Stage 1 lesions are generally treated with observation only.

Refers to lesions that are causing pain or some form of disability. If a lesion has weakened the structure of the bone such that fracture may occur, the lesion would be considered a Stage 2 lesion.

Refers to lesions that are large, have broken into the soft tissues, or have caused a pathologic

Aggressive fracture. These lesions are usually prone to local recurrence and have the potential of causing a major problem for the patient.

Correct Answer: Stage 2

 

 

 

 

 

 

Figure 1 Figure 2 Figure 3 Figure 4

 

 

 

Figure 5 Figure 6 Figure 7

A 10-year-old boy has immediate pain after throwing a baseball. He is unable to elevate his arm due to pain. The child has no medical problems. An anteroposterior radiograph of the shoulder is shown in Figure 1. The most likely diagnosis based on the history and radiograph is:

 

1) Giant cell tumor of bone

3) Ewingâs tumor

2) Aneursymal bone cyst

5) Osteosarcoma

4) Unicameral bone cyst

 

The plain radiograph shows a lytic metaphyseal lesion with symmetric expansion and thinned cortices. A minimally displaced fracture is also present. This is the classic appearance or a unicameral bone cyst.

Radiographic features of a unicameral bone cyst (Figure 2):

 

 

Metaphyseal Purely lytic

 

 

Symmetric expansion (approximately equal in width to the growth plate) Thinned but intact cortices

The most common location of a unicameral bone cyst is the proximal humerus, followed by the proximal femur. The other possible answers have different radiographic and clinical features:

Giant cell tumor:

Aneurysmal bone cyst:

Ewingâs tumor:

Uncommon in children with open physes; lesions start in the metaphysis and extend into the epiphysis often abutting the subchondral bone.

Metaphyseal, expansile lesion that is eccentric. The lesion often extends into the soft tissues and is covered by a thin shell of periosteal bone.

Lytic, destructive lesion, often with periosteal reaction.

Osteosarcoma: Metaphyseal lesion with mixed pattern of bone destruction and bone formation.

Correct Answer: Unicameral bone cyst

 

 

Figure 1 Figure 2

A 10-year-old boy has immediate pain after throwing a baseball. He is unable to elevate his arm due to pain. The child has no medical problems. An anteroposterior radiograph of the shoulder is shown in Figure 1. The most appropriate treatment for this condition is:

 

1) Preoperative chemotherapy followed by wide resection

3) Wide resection and allograft reconstruction

2) Chemotherapy and external beam irradiation

5) Curettage and augmentation with methyl methacrylate

4) Methyl prednisolone acetate injection

 

To answer this question, one must be able to establish the diagnosis from the radiographs. The diagnosis is unicameral bone cyst.

The plain radiograph shows a lytic metaphyseal lesion with symmetric expansion and thinned cortices. There is also a minimally displaced fracture. This is the classic appearance of a unicameral bone cyst.

Radiographic features of a unicameral bone cyst (Fig. 2):

 

 

Metaphyseal Purely lytic

 

 

Symmetric expansion (about equal in width to the growth plate Thinned, but intact, cortices

The treatment of unicameral bone cysts is aspiration of the cyst and injection with methyl prednisolone acetate (150 mg to 200 mg). Many cysts resolve with a single injection, whereas some require more than one aspiration/injection.

Correct Answer: Methyl prednisolone acetate injection